Slot System
Featured Buckets
Featured Buckets Admin

When does acute pyelonephritis require imaging?

Article Type
Changed
Thu, 08/01/2019 - 07:54
Display Headline
When does acute pyelonephritis require imaging?

A previously healthy 44-year-old woman presents to the emergency department with 1 day of fever, flank pain, dysuria, and persistent nausea and vomiting. Her temperature is 38.7°C (101.7°F), heart rate 102 beats per minute, and blood pressure 120/70 mm Hg. She has costovertebral angle tenderness. Laboratory testing reveals mild leukocytosis and a normal serum creatinine level; urinalysis shows leukocytes, as well as leukocyte esterase and nitrites. She has no personal or family history of nephrolithiasis. Urine cultures are obtained, and she is started on intravenous antibiotics and intravenous hydration to treat pyelonephritis.

Is imaging indicated at this point? And if so, which study is recommended?

KEY FEATURES

Acute pyelonephritis, infection of the renal parenchyma and collecting system, most often results from an ascending infection of the lower urinary tract. It is estimated to account for 250,000 office visits and 200,000 hospital admissions each year in the United States.1

Lower urinary tract symptoms such as urinary frequency, urgency, and dysuria accompanied by fever, nausea, vomiting, and flank pain raise suspicion for acute pyelonephritis. Flank pain is a key, nearly universal feature of upper urinary tract infection in patients without diabetes, though it may be absent in up to 50% of patients with diabetes.2

Additional findings include costovertebral angle tenderness on physical examination and leukocytosis, pyuria, and bacteriuria on laboratory studies.

PREDICTING THE NEED FOR EARLY IMAGING

Figure 1. Pathway for considering imaging in acute pyelonephritis. The recommended imaging study is computed tomography of the abdomen and pelvis with contrast, or computed tomography without and with contrast. The choice may be tailored to the patient’s
Figure 1. Pathway for considering imaging in acute pyelonephritis. The recommended imaging study is computed tomography of the abdomen and pelvis with contrast, or computed tomography without and with contrast. The choice may be tailored to the patient’s clinical condition.
The primary goal of imaging in acute pyelo­nephritis is to identify significant complications (eg, gas-forming infection, abscess formation, urinary obstruction) that may necessitate a change in management. Risk of complications is higher in patients with diabetes, compromised immunity, a recent urologic procedure, or a history of urolithiasis. Patients without these risk factors usually do not need imaging unless fever or leukocytosis persists 72 hours after the start of antibiotics. Figure 1 offers guidance on deciding when imaging is needed in these patients.

Though guidelines state that imaging is inappropriate in most patients with pyeloneph­ritis,2–4 it is nevertheless often done for diagnosis or identification of complications, which have been reported in more than two-thirds of patients.2–4

Acute pyelonephritis is generally classified as complicated or uncomplicated, though different definitions exist with regard to these classifications. The American College of Radiology’s Appropriateness Criteria2 consider patients with diabetes, immune compromise, a history of urolithiasis, or anatomic abnormality to be at highest risk for complications, and therefore recommend early imaging to assess for hydronephrosis, pyonephrosis, emphysematous pyelonephritis, and intrinsic or perinephric abscess.2

A clinical rule for predicting the need for imaging in acute pyelonephritis was developed and validated in an emergency department population in the Netherlands.3 The study suggested that restricting early imaging to patients with a history of urolithiasis, a urine pH of 7.0 or higher, or renal insufficiency—defined as a glomerular filtration rate (GFR) of 40 mL/min/1.73m2 or lower as estimated by the Modification of Diet in Renal Disease formula—would provide a negative predictive value of 94% to 100% for detection of an urgent urologic disorder (pyonephrosis, renal abscess, or urolithiasis). This high negative predictive value highlights that an absence of these signs and symptoms can safely identify patients who do not need renal imaging.

The positive predictive value was less useful, as only 5% to 23% of patients who had at least 1 risk factor went on to have urgent urologic risk factors.3

Implementation of this prediction rule would have resulted in a relative reduction in imaging of 40% and an absolute reduction of 28%. Of note, use of reduced GFR in this prediction rule is not clearly validated for patients with chronic kidney disease, as the previous GFR for most patients in this study was unknown.3

Based on these data, initial imaging is recommended in patients with diabetes, immune compromise, a history of urolithiasis, anatomic abnormality, a urine pH 7.0 or higher, or a GFR 40 mL/min or lower in a patient with no history of significant renal dysfunction. Early imaging would also be reasonable in patients with a complex clinical presentation, early recurrence of symptoms after treatment, clinical decompensation, or critical illness.

 

 

TREATMENT FAILURE

In a retrospective review of 62 patients hospitalized for acute renal infection, Soulen et al5 found that the most reliable indicator of complicated acute pyelonephritis was the persistence of fever and leukocytosis at 72 hours. And another small prospective study of patients with uncomplicated pyelonephritis reported a time to defervescence of no more than 4 days.6

In accordance with the Appropriateness Criteria2 and based on the best available evidence, imaging is recommended in all patients who remain febrile or have persistent leukocytosis after 72 hours of antibiotic therapy. In such cases, there should be high suspicion for a complication requiring treatment.

OPTIONS FOR IMAGING

Computed tomography

Computed tomography (CT) of the abdomen and pelvis with contrast is considered the study of choice in complicated acute pyelonephritis. CT can detect focal parenchymal abnormalities, emphysematous changes, and anatomic anomalies, and can also define the extent of disease. It can also detect perinephric fluid collections and abscesses that necessitate a change in management.2,5

A retrospective study in 2017 found that contrast-enhanced CT done without the usual noncontrast and excretory phases had an accuracy of 90% to 92% for pyelonephritis and 96% to 99% for urolithiasis, suggesting that reduction in radiation exposure through use of only the contrast-enhanced phase of CT imaging may be reasonable.7

Magnetic resonance imaging

Magnetic resonance imaging (MRI) is increasingly acknowledged as effective in the evaluation of renal pathology, including the diagnosis of pyelonephritis; but it lacks the level of evidence that CT provides for detecting renal abscesses, calculi, and emphysematous pyelonephritis.2,8,9

Though it is more costly and time-consuming than CT with contrast enhancement, MRI is nevertheless the imaging study of choice if iodinated contrast or ionizing radiation must be avoided.

MRI typically involves a precontrast phase and a gadolinium contrast-enhanced phase, though there are data to support diffusion-weighted MRI when exposure to gadolinium poses a risk to the patient, such as in pregnancy or renal impairment (particularly when the estimated GFR is < 30 mL/min/1.73 m2).10

Ultrasonography

Conventional ultrasonography is appealing due to its relatively low cost, its availability and portability, and the lack of radiation and contrast exposure. It is most helpful in detecting hydronephrosis and pyonephrosis rather than intrarenal or perinephric abscess.2,9

Color and power Doppler ultrasonography may improve testing characteristics but not to the level of CT; in one study, sensitivity for detection of pyelonephritis was 33.3% with ultrasonography vs 81.0% with CT.11

Recent studies of ultrasonography with contrast enhancement show promising results,2 and it may ultimately prove to have a similar efficacy with lower risk for patients, but this has not been validated in large studies, and its availability remains limited.

Ultrasonography should be considered for patients in whom obstruction (with resulting hydronephrosis or pyonephrosis) is a primary concern, particularly when contrast exposure or radiation is contraindicated and MRI is unavailable.2

Abdominal radiography

While emphysematous pyelonephritis or a large staghorn calculus may be seen on abdominal radiography, it is not recommended for the assessment of complications in acute pyelonephritis because it lacks sensitivity.2

RETURN TO THE CASE SCENARIO

The patient in our case scenario meets the clinical criteria for uncomplicated pyelo­nephritis and is therefore not a candidate for imaging. Intravenous antibiotics should be started and should lead to rapid improvement in her condition.

Acknowledgment: The authors would like to thank Dr. Lisa Blacklock for her review of the radiology section of this paper.

References
  1. Foxman B, Klemstine KL, Brown PD. Acute pyelonephritis in US hospitals in 1997: hospitalization and in-hospital mortality. Ann Epidemiol 2003; 13(2):144–150. pmid:12559674
  2. Expert Panel on Urologic Imaging: Nikolaidis P, Dogra VS, Goldfarb S, et al. ACR appropriateness criteria acute pyelonephritis. J Am Coll Radiol 2018; 15(11S):S232–S239. doi:10.1016/j.jacr.2018.09.011
  3. van Nieuwkoop C, Hoppe BP, Bonten TN, et al. Predicting the need for radiologic imaging in adults with febrile urinary tract infection. Clin Infect Dis 2010; 51(11):1266–1272. doi:10.1086/657071
  4. Kim Y, Seo MR, Kim SJ, et al. Usefulness of blood cultures and radiologic imaging studies in the management of patients with community-acquired acute pyelonephritis. Infect Chemother 2017; 49(1):22–30. doi:10.3947/ic.2017.49.1.22
  5. Soulen MC, Fishman EK, Goldman SM, Gatewood OM. Bacterial renal infection: role of CT. Radiology 1989; 171(3):703–707. doi:10.1148/radiology.171.3.2655002
  6. June CH, Browning MD, Smith LP, et al. Ultrasonography and computed tomography in severe urinary tract infection. Arch Intern Med 1985; 145(5):841–845. pmid:3888134
  7. Taniguchi LS, Torres US, Souza SM, Torres LR, D’Ippolito G. Are the unenhanced and excretory CT phases necessary for the evaluation of acute pyelonephritis? Acta Radiol 2017; 58(5):634–640. doi:10.1177/0284185116665424
  8. Rathod SB, Kumbhar SS, Nanivadekar A, Aman K. Role of diffusion-weighted MRI in acute pyelonephritis: a prospective study. Acta Radiol 2015; 56(2):244–249. doi:10.1177/0284185114520862
  9. Stunell H, Buckley O, Feeney J, Geoghegan T, Browne RF, Torreggiani WC. Imaging of acute pyelonephritis in the adult. Eur Radiol 2007; 17(7):1820–1828.
  10. American College of Radiology. ACR Manual on Contrast Media. www.acr.org/clinical-resources/contrast-manual. Accessed June 19, 2019.
  11. Yoo JM, Koh JS, Han CH, et al. Diagnosing acute pyelonephritis with CT, Tc-DMSA SPECT, and Doppler ultrasound: a comparative study. Korean J Urol 2010; 51(4):260–265. doi:10.4111/kju.2010.51.4.260
Article PDF
Author and Disclosure Information

Mary E. Lacy, MD
Assistant Professor, Department of Internal Medicine, University of New Mexico School of Medicine, Albuquerque, NM

Navneet Sidhu, MD
Department Head, Hospitalist Medicine, Langley Memorial Hospital, Fraser Health, Langley, BC, Canada

Justin Miller, MD
Assistant Professor, Department of Internal Medicine, University of New Mexico School of Medicine, Albuquerque, NM

Address: Mary E. Lacy, MD, Department of Internal Medicine, University of New Mexico School of Medicine, MSC 10-550, University of NM SOM, Albuquerque, NM 87131; melacy@salud.unm.edu

Issue
Cleveland Clinic Journal of Medicine - 86(8)
Publications
Topics
Page Number
515-517
Legacy Keywords
acute pyelonephritis, urinary tract infection, UTI, pelvic inflammatory disease, PID, kidney infection, renal infection, pyuria, bactiuria, flank pain, imaging, computed tomography, CT, magnetic resonance imaging, MRI, ultrasonography, radiography, antibiotics, Mary Lacy, Navneet Sidhu, Justin Miller
Sections
Author and Disclosure Information

Mary E. Lacy, MD
Assistant Professor, Department of Internal Medicine, University of New Mexico School of Medicine, Albuquerque, NM

Navneet Sidhu, MD
Department Head, Hospitalist Medicine, Langley Memorial Hospital, Fraser Health, Langley, BC, Canada

Justin Miller, MD
Assistant Professor, Department of Internal Medicine, University of New Mexico School of Medicine, Albuquerque, NM

Address: Mary E. Lacy, MD, Department of Internal Medicine, University of New Mexico School of Medicine, MSC 10-550, University of NM SOM, Albuquerque, NM 87131; melacy@salud.unm.edu

Author and Disclosure Information

Mary E. Lacy, MD
Assistant Professor, Department of Internal Medicine, University of New Mexico School of Medicine, Albuquerque, NM

Navneet Sidhu, MD
Department Head, Hospitalist Medicine, Langley Memorial Hospital, Fraser Health, Langley, BC, Canada

Justin Miller, MD
Assistant Professor, Department of Internal Medicine, University of New Mexico School of Medicine, Albuquerque, NM

Address: Mary E. Lacy, MD, Department of Internal Medicine, University of New Mexico School of Medicine, MSC 10-550, University of NM SOM, Albuquerque, NM 87131; melacy@salud.unm.edu

Article PDF
Article PDF
Related Articles

A previously healthy 44-year-old woman presents to the emergency department with 1 day of fever, flank pain, dysuria, and persistent nausea and vomiting. Her temperature is 38.7°C (101.7°F), heart rate 102 beats per minute, and blood pressure 120/70 mm Hg. She has costovertebral angle tenderness. Laboratory testing reveals mild leukocytosis and a normal serum creatinine level; urinalysis shows leukocytes, as well as leukocyte esterase and nitrites. She has no personal or family history of nephrolithiasis. Urine cultures are obtained, and she is started on intravenous antibiotics and intravenous hydration to treat pyelonephritis.

Is imaging indicated at this point? And if so, which study is recommended?

KEY FEATURES

Acute pyelonephritis, infection of the renal parenchyma and collecting system, most often results from an ascending infection of the lower urinary tract. It is estimated to account for 250,000 office visits and 200,000 hospital admissions each year in the United States.1

Lower urinary tract symptoms such as urinary frequency, urgency, and dysuria accompanied by fever, nausea, vomiting, and flank pain raise suspicion for acute pyelonephritis. Flank pain is a key, nearly universal feature of upper urinary tract infection in patients without diabetes, though it may be absent in up to 50% of patients with diabetes.2

Additional findings include costovertebral angle tenderness on physical examination and leukocytosis, pyuria, and bacteriuria on laboratory studies.

PREDICTING THE NEED FOR EARLY IMAGING

Figure 1. Pathway for considering imaging in acute pyelonephritis. The recommended imaging study is computed tomography of the abdomen and pelvis with contrast, or computed tomography without and with contrast. The choice may be tailored to the patient’s
Figure 1. Pathway for considering imaging in acute pyelonephritis. The recommended imaging study is computed tomography of the abdomen and pelvis with contrast, or computed tomography without and with contrast. The choice may be tailored to the patient’s clinical condition.
The primary goal of imaging in acute pyelo­nephritis is to identify significant complications (eg, gas-forming infection, abscess formation, urinary obstruction) that may necessitate a change in management. Risk of complications is higher in patients with diabetes, compromised immunity, a recent urologic procedure, or a history of urolithiasis. Patients without these risk factors usually do not need imaging unless fever or leukocytosis persists 72 hours after the start of antibiotics. Figure 1 offers guidance on deciding when imaging is needed in these patients.

Though guidelines state that imaging is inappropriate in most patients with pyeloneph­ritis,2–4 it is nevertheless often done for diagnosis or identification of complications, which have been reported in more than two-thirds of patients.2–4

Acute pyelonephritis is generally classified as complicated or uncomplicated, though different definitions exist with regard to these classifications. The American College of Radiology’s Appropriateness Criteria2 consider patients with diabetes, immune compromise, a history of urolithiasis, or anatomic abnormality to be at highest risk for complications, and therefore recommend early imaging to assess for hydronephrosis, pyonephrosis, emphysematous pyelonephritis, and intrinsic or perinephric abscess.2

A clinical rule for predicting the need for imaging in acute pyelonephritis was developed and validated in an emergency department population in the Netherlands.3 The study suggested that restricting early imaging to patients with a history of urolithiasis, a urine pH of 7.0 or higher, or renal insufficiency—defined as a glomerular filtration rate (GFR) of 40 mL/min/1.73m2 or lower as estimated by the Modification of Diet in Renal Disease formula—would provide a negative predictive value of 94% to 100% for detection of an urgent urologic disorder (pyonephrosis, renal abscess, or urolithiasis). This high negative predictive value highlights that an absence of these signs and symptoms can safely identify patients who do not need renal imaging.

The positive predictive value was less useful, as only 5% to 23% of patients who had at least 1 risk factor went on to have urgent urologic risk factors.3

Implementation of this prediction rule would have resulted in a relative reduction in imaging of 40% and an absolute reduction of 28%. Of note, use of reduced GFR in this prediction rule is not clearly validated for patients with chronic kidney disease, as the previous GFR for most patients in this study was unknown.3

Based on these data, initial imaging is recommended in patients with diabetes, immune compromise, a history of urolithiasis, anatomic abnormality, a urine pH 7.0 or higher, or a GFR 40 mL/min or lower in a patient with no history of significant renal dysfunction. Early imaging would also be reasonable in patients with a complex clinical presentation, early recurrence of symptoms after treatment, clinical decompensation, or critical illness.

 

 

TREATMENT FAILURE

In a retrospective review of 62 patients hospitalized for acute renal infection, Soulen et al5 found that the most reliable indicator of complicated acute pyelonephritis was the persistence of fever and leukocytosis at 72 hours. And another small prospective study of patients with uncomplicated pyelonephritis reported a time to defervescence of no more than 4 days.6

In accordance with the Appropriateness Criteria2 and based on the best available evidence, imaging is recommended in all patients who remain febrile or have persistent leukocytosis after 72 hours of antibiotic therapy. In such cases, there should be high suspicion for a complication requiring treatment.

OPTIONS FOR IMAGING

Computed tomography

Computed tomography (CT) of the abdomen and pelvis with contrast is considered the study of choice in complicated acute pyelonephritis. CT can detect focal parenchymal abnormalities, emphysematous changes, and anatomic anomalies, and can also define the extent of disease. It can also detect perinephric fluid collections and abscesses that necessitate a change in management.2,5

A retrospective study in 2017 found that contrast-enhanced CT done without the usual noncontrast and excretory phases had an accuracy of 90% to 92% for pyelonephritis and 96% to 99% for urolithiasis, suggesting that reduction in radiation exposure through use of only the contrast-enhanced phase of CT imaging may be reasonable.7

Magnetic resonance imaging

Magnetic resonance imaging (MRI) is increasingly acknowledged as effective in the evaluation of renal pathology, including the diagnosis of pyelonephritis; but it lacks the level of evidence that CT provides for detecting renal abscesses, calculi, and emphysematous pyelonephritis.2,8,9

Though it is more costly and time-consuming than CT with contrast enhancement, MRI is nevertheless the imaging study of choice if iodinated contrast or ionizing radiation must be avoided.

MRI typically involves a precontrast phase and a gadolinium contrast-enhanced phase, though there are data to support diffusion-weighted MRI when exposure to gadolinium poses a risk to the patient, such as in pregnancy or renal impairment (particularly when the estimated GFR is < 30 mL/min/1.73 m2).10

Ultrasonography

Conventional ultrasonography is appealing due to its relatively low cost, its availability and portability, and the lack of radiation and contrast exposure. It is most helpful in detecting hydronephrosis and pyonephrosis rather than intrarenal or perinephric abscess.2,9

Color and power Doppler ultrasonography may improve testing characteristics but not to the level of CT; in one study, sensitivity for detection of pyelonephritis was 33.3% with ultrasonography vs 81.0% with CT.11

Recent studies of ultrasonography with contrast enhancement show promising results,2 and it may ultimately prove to have a similar efficacy with lower risk for patients, but this has not been validated in large studies, and its availability remains limited.

Ultrasonography should be considered for patients in whom obstruction (with resulting hydronephrosis or pyonephrosis) is a primary concern, particularly when contrast exposure or radiation is contraindicated and MRI is unavailable.2

Abdominal radiography

While emphysematous pyelonephritis or a large staghorn calculus may be seen on abdominal radiography, it is not recommended for the assessment of complications in acute pyelonephritis because it lacks sensitivity.2

RETURN TO THE CASE SCENARIO

The patient in our case scenario meets the clinical criteria for uncomplicated pyelo­nephritis and is therefore not a candidate for imaging. Intravenous antibiotics should be started and should lead to rapid improvement in her condition.

Acknowledgment: The authors would like to thank Dr. Lisa Blacklock for her review of the radiology section of this paper.

A previously healthy 44-year-old woman presents to the emergency department with 1 day of fever, flank pain, dysuria, and persistent nausea and vomiting. Her temperature is 38.7°C (101.7°F), heart rate 102 beats per minute, and blood pressure 120/70 mm Hg. She has costovertebral angle tenderness. Laboratory testing reveals mild leukocytosis and a normal serum creatinine level; urinalysis shows leukocytes, as well as leukocyte esterase and nitrites. She has no personal or family history of nephrolithiasis. Urine cultures are obtained, and she is started on intravenous antibiotics and intravenous hydration to treat pyelonephritis.

Is imaging indicated at this point? And if so, which study is recommended?

KEY FEATURES

Acute pyelonephritis, infection of the renal parenchyma and collecting system, most often results from an ascending infection of the lower urinary tract. It is estimated to account for 250,000 office visits and 200,000 hospital admissions each year in the United States.1

Lower urinary tract symptoms such as urinary frequency, urgency, and dysuria accompanied by fever, nausea, vomiting, and flank pain raise suspicion for acute pyelonephritis. Flank pain is a key, nearly universal feature of upper urinary tract infection in patients without diabetes, though it may be absent in up to 50% of patients with diabetes.2

Additional findings include costovertebral angle tenderness on physical examination and leukocytosis, pyuria, and bacteriuria on laboratory studies.

PREDICTING THE NEED FOR EARLY IMAGING

Figure 1. Pathway for considering imaging in acute pyelonephritis. The recommended imaging study is computed tomography of the abdomen and pelvis with contrast, or computed tomography without and with contrast. The choice may be tailored to the patient’s
Figure 1. Pathway for considering imaging in acute pyelonephritis. The recommended imaging study is computed tomography of the abdomen and pelvis with contrast, or computed tomography without and with contrast. The choice may be tailored to the patient’s clinical condition.
The primary goal of imaging in acute pyelo­nephritis is to identify significant complications (eg, gas-forming infection, abscess formation, urinary obstruction) that may necessitate a change in management. Risk of complications is higher in patients with diabetes, compromised immunity, a recent urologic procedure, or a history of urolithiasis. Patients without these risk factors usually do not need imaging unless fever or leukocytosis persists 72 hours after the start of antibiotics. Figure 1 offers guidance on deciding when imaging is needed in these patients.

Though guidelines state that imaging is inappropriate in most patients with pyeloneph­ritis,2–4 it is nevertheless often done for diagnosis or identification of complications, which have been reported in more than two-thirds of patients.2–4

Acute pyelonephritis is generally classified as complicated or uncomplicated, though different definitions exist with regard to these classifications. The American College of Radiology’s Appropriateness Criteria2 consider patients with diabetes, immune compromise, a history of urolithiasis, or anatomic abnormality to be at highest risk for complications, and therefore recommend early imaging to assess for hydronephrosis, pyonephrosis, emphysematous pyelonephritis, and intrinsic or perinephric abscess.2

A clinical rule for predicting the need for imaging in acute pyelonephritis was developed and validated in an emergency department population in the Netherlands.3 The study suggested that restricting early imaging to patients with a history of urolithiasis, a urine pH of 7.0 or higher, or renal insufficiency—defined as a glomerular filtration rate (GFR) of 40 mL/min/1.73m2 or lower as estimated by the Modification of Diet in Renal Disease formula—would provide a negative predictive value of 94% to 100% for detection of an urgent urologic disorder (pyonephrosis, renal abscess, or urolithiasis). This high negative predictive value highlights that an absence of these signs and symptoms can safely identify patients who do not need renal imaging.

The positive predictive value was less useful, as only 5% to 23% of patients who had at least 1 risk factor went on to have urgent urologic risk factors.3

Implementation of this prediction rule would have resulted in a relative reduction in imaging of 40% and an absolute reduction of 28%. Of note, use of reduced GFR in this prediction rule is not clearly validated for patients with chronic kidney disease, as the previous GFR for most patients in this study was unknown.3

Based on these data, initial imaging is recommended in patients with diabetes, immune compromise, a history of urolithiasis, anatomic abnormality, a urine pH 7.0 or higher, or a GFR 40 mL/min or lower in a patient with no history of significant renal dysfunction. Early imaging would also be reasonable in patients with a complex clinical presentation, early recurrence of symptoms after treatment, clinical decompensation, or critical illness.

 

 

TREATMENT FAILURE

In a retrospective review of 62 patients hospitalized for acute renal infection, Soulen et al5 found that the most reliable indicator of complicated acute pyelonephritis was the persistence of fever and leukocytosis at 72 hours. And another small prospective study of patients with uncomplicated pyelonephritis reported a time to defervescence of no more than 4 days.6

In accordance with the Appropriateness Criteria2 and based on the best available evidence, imaging is recommended in all patients who remain febrile or have persistent leukocytosis after 72 hours of antibiotic therapy. In such cases, there should be high suspicion for a complication requiring treatment.

OPTIONS FOR IMAGING

Computed tomography

Computed tomography (CT) of the abdomen and pelvis with contrast is considered the study of choice in complicated acute pyelonephritis. CT can detect focal parenchymal abnormalities, emphysematous changes, and anatomic anomalies, and can also define the extent of disease. It can also detect perinephric fluid collections and abscesses that necessitate a change in management.2,5

A retrospective study in 2017 found that contrast-enhanced CT done without the usual noncontrast and excretory phases had an accuracy of 90% to 92% for pyelonephritis and 96% to 99% for urolithiasis, suggesting that reduction in radiation exposure through use of only the contrast-enhanced phase of CT imaging may be reasonable.7

Magnetic resonance imaging

Magnetic resonance imaging (MRI) is increasingly acknowledged as effective in the evaluation of renal pathology, including the diagnosis of pyelonephritis; but it lacks the level of evidence that CT provides for detecting renal abscesses, calculi, and emphysematous pyelonephritis.2,8,9

Though it is more costly and time-consuming than CT with contrast enhancement, MRI is nevertheless the imaging study of choice if iodinated contrast or ionizing radiation must be avoided.

MRI typically involves a precontrast phase and a gadolinium contrast-enhanced phase, though there are data to support diffusion-weighted MRI when exposure to gadolinium poses a risk to the patient, such as in pregnancy or renal impairment (particularly when the estimated GFR is < 30 mL/min/1.73 m2).10

Ultrasonography

Conventional ultrasonography is appealing due to its relatively low cost, its availability and portability, and the lack of radiation and contrast exposure. It is most helpful in detecting hydronephrosis and pyonephrosis rather than intrarenal or perinephric abscess.2,9

Color and power Doppler ultrasonography may improve testing characteristics but not to the level of CT; in one study, sensitivity for detection of pyelonephritis was 33.3% with ultrasonography vs 81.0% with CT.11

Recent studies of ultrasonography with contrast enhancement show promising results,2 and it may ultimately prove to have a similar efficacy with lower risk for patients, but this has not been validated in large studies, and its availability remains limited.

Ultrasonography should be considered for patients in whom obstruction (with resulting hydronephrosis or pyonephrosis) is a primary concern, particularly when contrast exposure or radiation is contraindicated and MRI is unavailable.2

Abdominal radiography

While emphysematous pyelonephritis or a large staghorn calculus may be seen on abdominal radiography, it is not recommended for the assessment of complications in acute pyelonephritis because it lacks sensitivity.2

RETURN TO THE CASE SCENARIO

The patient in our case scenario meets the clinical criteria for uncomplicated pyelo­nephritis and is therefore not a candidate for imaging. Intravenous antibiotics should be started and should lead to rapid improvement in her condition.

Acknowledgment: The authors would like to thank Dr. Lisa Blacklock for her review of the radiology section of this paper.

References
  1. Foxman B, Klemstine KL, Brown PD. Acute pyelonephritis in US hospitals in 1997: hospitalization and in-hospital mortality. Ann Epidemiol 2003; 13(2):144–150. pmid:12559674
  2. Expert Panel on Urologic Imaging: Nikolaidis P, Dogra VS, Goldfarb S, et al. ACR appropriateness criteria acute pyelonephritis. J Am Coll Radiol 2018; 15(11S):S232–S239. doi:10.1016/j.jacr.2018.09.011
  3. van Nieuwkoop C, Hoppe BP, Bonten TN, et al. Predicting the need for radiologic imaging in adults with febrile urinary tract infection. Clin Infect Dis 2010; 51(11):1266–1272. doi:10.1086/657071
  4. Kim Y, Seo MR, Kim SJ, et al. Usefulness of blood cultures and radiologic imaging studies in the management of patients with community-acquired acute pyelonephritis. Infect Chemother 2017; 49(1):22–30. doi:10.3947/ic.2017.49.1.22
  5. Soulen MC, Fishman EK, Goldman SM, Gatewood OM. Bacterial renal infection: role of CT. Radiology 1989; 171(3):703–707. doi:10.1148/radiology.171.3.2655002
  6. June CH, Browning MD, Smith LP, et al. Ultrasonography and computed tomography in severe urinary tract infection. Arch Intern Med 1985; 145(5):841–845. pmid:3888134
  7. Taniguchi LS, Torres US, Souza SM, Torres LR, D’Ippolito G. Are the unenhanced and excretory CT phases necessary for the evaluation of acute pyelonephritis? Acta Radiol 2017; 58(5):634–640. doi:10.1177/0284185116665424
  8. Rathod SB, Kumbhar SS, Nanivadekar A, Aman K. Role of diffusion-weighted MRI in acute pyelonephritis: a prospective study. Acta Radiol 2015; 56(2):244–249. doi:10.1177/0284185114520862
  9. Stunell H, Buckley O, Feeney J, Geoghegan T, Browne RF, Torreggiani WC. Imaging of acute pyelonephritis in the adult. Eur Radiol 2007; 17(7):1820–1828.
  10. American College of Radiology. ACR Manual on Contrast Media. www.acr.org/clinical-resources/contrast-manual. Accessed June 19, 2019.
  11. Yoo JM, Koh JS, Han CH, et al. Diagnosing acute pyelonephritis with CT, Tc-DMSA SPECT, and Doppler ultrasound: a comparative study. Korean J Urol 2010; 51(4):260–265. doi:10.4111/kju.2010.51.4.260
References
  1. Foxman B, Klemstine KL, Brown PD. Acute pyelonephritis in US hospitals in 1997: hospitalization and in-hospital mortality. Ann Epidemiol 2003; 13(2):144–150. pmid:12559674
  2. Expert Panel on Urologic Imaging: Nikolaidis P, Dogra VS, Goldfarb S, et al. ACR appropriateness criteria acute pyelonephritis. J Am Coll Radiol 2018; 15(11S):S232–S239. doi:10.1016/j.jacr.2018.09.011
  3. van Nieuwkoop C, Hoppe BP, Bonten TN, et al. Predicting the need for radiologic imaging in adults with febrile urinary tract infection. Clin Infect Dis 2010; 51(11):1266–1272. doi:10.1086/657071
  4. Kim Y, Seo MR, Kim SJ, et al. Usefulness of blood cultures and radiologic imaging studies in the management of patients with community-acquired acute pyelonephritis. Infect Chemother 2017; 49(1):22–30. doi:10.3947/ic.2017.49.1.22
  5. Soulen MC, Fishman EK, Goldman SM, Gatewood OM. Bacterial renal infection: role of CT. Radiology 1989; 171(3):703–707. doi:10.1148/radiology.171.3.2655002
  6. June CH, Browning MD, Smith LP, et al. Ultrasonography and computed tomography in severe urinary tract infection. Arch Intern Med 1985; 145(5):841–845. pmid:3888134
  7. Taniguchi LS, Torres US, Souza SM, Torres LR, D’Ippolito G. Are the unenhanced and excretory CT phases necessary for the evaluation of acute pyelonephritis? Acta Radiol 2017; 58(5):634–640. doi:10.1177/0284185116665424
  8. Rathod SB, Kumbhar SS, Nanivadekar A, Aman K. Role of diffusion-weighted MRI in acute pyelonephritis: a prospective study. Acta Radiol 2015; 56(2):244–249. doi:10.1177/0284185114520862
  9. Stunell H, Buckley O, Feeney J, Geoghegan T, Browne RF, Torreggiani WC. Imaging of acute pyelonephritis in the adult. Eur Radiol 2007; 17(7):1820–1828.
  10. American College of Radiology. ACR Manual on Contrast Media. www.acr.org/clinical-resources/contrast-manual. Accessed June 19, 2019.
  11. Yoo JM, Koh JS, Han CH, et al. Diagnosing acute pyelonephritis with CT, Tc-DMSA SPECT, and Doppler ultrasound: a comparative study. Korean J Urol 2010; 51(4):260–265. doi:10.4111/kju.2010.51.4.260
Issue
Cleveland Clinic Journal of Medicine - 86(8)
Issue
Cleveland Clinic Journal of Medicine - 86(8)
Page Number
515-517
Page Number
515-517
Publications
Publications
Topics
Article Type
Display Headline
When does acute pyelonephritis require imaging?
Display Headline
When does acute pyelonephritis require imaging?
Legacy Keywords
acute pyelonephritis, urinary tract infection, UTI, pelvic inflammatory disease, PID, kidney infection, renal infection, pyuria, bactiuria, flank pain, imaging, computed tomography, CT, magnetic resonance imaging, MRI, ultrasonography, radiography, antibiotics, Mary Lacy, Navneet Sidhu, Justin Miller
Legacy Keywords
acute pyelonephritis, urinary tract infection, UTI, pelvic inflammatory disease, PID, kidney infection, renal infection, pyuria, bactiuria, flank pain, imaging, computed tomography, CT, magnetic resonance imaging, MRI, ultrasonography, radiography, antibiotics, Mary Lacy, Navneet Sidhu, Justin Miller
Sections
Disallow All Ads
Content Gating
No Gating (article Unlocked/Free)
Alternative CME
Disqus Comments
Default
Gate On Date
Mon, 07/29/2019 - 11:15
Un-Gate On Date
Mon, 07/29/2019 - 11:15
Use ProPublica
CFC Schedule Remove Status
Mon, 07/29/2019 - 11:15
Hide sidebar & use full width
render the right sidebar.
Article PDF Media

Is neuroimaging necessary to evaluate syncope?

Article Type
Changed
Mon, 04/01/2019 - 08:53
Display Headline
Is neuroimaging necessary to evaluate syncope?

A 40-year-old woman with a history of hypertension, who was recently started on a diuretic, presents to the emergency department after a witnessed syncopal event. She reports a prodrome of lightheadedness, nausea, and darkening of her vision that occurred a few seconds after standing, followed by loss of consciousness. She had a complete, spontaneous recovery after 10 seconds, but upon arousal she noticed she had lost bladder control.

Her blood pressure is 120/80 mm Hg supine, 110/70 mm Hg sitting, and 90/60 mm Hg standing. She has no focal neurologic deficits. The cardiac examination is normal, without murmurs, and electrocardiography shows sinus tachycardia (heart rate 110 bpm) without other abnormalities. Results of laboratory testing are unremarkable.

Should you order neuroimaging to evaluate for syncope?

DEFINITIONS, CLASSIFICATIONS

Syncope is an abrupt loss of consciousness due to transient global cerebral hypoperfusion, with a concomitant loss of postural tone and rapid, spontaneous recovery.1 Recovery from syncope is characterized by immediate restoration of orientation and normal behavior, although the period after recovery may be accompanied by fatigue.2

The European Society of Cardiology2 has classified syncope into 3 main categories: reflex (neurally mediated) syncope, syncope due to orthostatic hypotension, and cardiac syncope. Determining the cause is critical, as this determines the prognosis.

KEYS TO THE EVALUATION

According to the 2017 American College of Cardiology/American Heart Association (ACC/AHA) and the 2009 European Society of Cardiology guidelines, the evaluation of syncope should include a thorough history, taken from the patient and witnesses, and a complete physical examination. This can identify the cause of syncope in up to 50% of cases and differentiate between cardiac and noncardiac causes. Features that point to cardiac syncope include age older than 60, male sex, known heart disease, brief prodrome, syncope during exertion or when supine, first syncopal event, family history of sudden cardiac death, and abnormal physical examination.1

Features that suggest noncardiac syncope are young age; syncope only when standing; recurrent syncope; a prodrome of nausea, vomiting, and a warm sensation; and triggers such as dehydration, pain, distressful stimulus, cough, laugh micturition, defecation, and swallowing.1

Electrocardiography should follow the history and physical examination. When done at presentation, electrocardiography is diagnostic in only about 5% of cases. However, given the importance of the diagnosis, it remains an essential part of the initial evaluation of syncope.3

If a clear cause of syncope is identified at this point, no further workup is needed, and the cause of syncope should be addressed.1 If the cause is still unclear, the ACC/AHA guidelines recommend further evaluation based on the clinical presentation and risk stratification.

 

 

WHEN TO PURSUE ADDITIONAL TESTING

Routine use of additional testing is costly; tests should be ordered on the basis of their potential diagnostic and prognostic value. Additional evaluation should follow a stepwise approach and can include targeted blood work, autonomic nerve evaluation, tilt-table testing, transthoracic echocardiography, stress testing, electrocardiographic monitoring, and electrophysiologic testing.1

Table 1. Criteria for high cardiac risk in syncope
If the initial evaluation indicates cardiac syncope (Table 1), evaluation with echocardiography has a class IIa recommendation (considered reasonable).1,2

Syncope is rarely a manifestation of neurologic disease, yet 11% to 58% of patients with a first episode of uncomplicated syncope undergo extensive neuroimaging with magnetic resonance imaging, computed tomography, electroencephalography (EEG), and carotid ultrasonography.4 Evidence suggests that routine neurologic testing is of limited value given its low diagnostic yield and high cost.

Epilepsy is the most common neurologic cause of loss of consciousness but is estimated to account for less than 5% of patients with syncope.5 A thorough and thoughtful neurologic history and examination is often enough to distinguish between syncope, convulsive syncope, epileptic convulsions, and pseudosyncope.

In syncope, the loss of consciousness usually occurs 30 seconds to several minutes after standing. It presents with or without a prodrome (warmth, palpitations, and diaphoresis) and can be relieved with supine positioning. True loss of consciousness usually lasts less than a minute and is accompanied by loss of postural tone, with little or no fatigue in the recovery period.6

Conversely, in convulsive syncope, the prodrome can include pallor and diaphoresis. Loss of consciousness lasts about 30 seconds but is accompanied by fixed gaze, upward eye deviation, nuchal rigidity, tonic spasms, myoclonic jerks, tonic-clonic convulsions, and oral automatisms.6

Pseudosyncope is characterized by a prodrome of lightheadedness, shortness of breath, chest pain, and tingling sensations, followed by episodes of apparent loss of consciousness that last longer than several minutes and occur multiple times a day. During these episodes, patients purposefully try to avoid trauma when they lose consciousness, and almost always keep their eyes closed, in contrast to syncopal episodes, when the eyes are open and glassy.7

ROLE OF ELECTROENCEPHALOGRAPHY

If the diagnosis remains unclear after the history and neurologic examination, EEG is recommended (class IIa, ie, reasonable, can be useful) during tilt-table testing, as it can help differentiate syncope, pseudosyncope, and epilepsy.1

In an epileptic convulsion, EEG shows epileptiform discharges, whereas in syncope, it shows diffuse brainwave slowing with delta waves and a flatline pattern. In pseudosyncope and psychogenic nonepileptic seizures, EEG shows normal activity.8

Routine EEG is not recommended if there are no specific neurologic signs of epilepsy or if the history and neurologic examination indicate syncope or pseudosyncope.1

Structural brain disease does not typically present with transient global cerebral hypoperfusion resulting in syncope, so magnetic resonance imaging and computed tomography have a low diagnostic yield. Studies have revealed that for the 11% to 58% of patients who undergo neuroimaging, it establishes a diagnosis in only 0.2% to 1%.9 For this reason and in view of their high cost, these imaging tests should not be routinely ordered in the evaluation of syncope.4,10 Similarly, carotid artery imaging should not be routinely ordered if there is no focal neurologic finding suggesting unilateral ischemia.10

CASE CONTINUED

In our 40-year-old patient, the history suggests dehydration, as she recently started taking a diuretic. Thus, laboratory testing is reasonable.

Loss of bladder control is often interpreted as a red flag for neurologic disease, but syncope can often present with urinary incontinence. Urinary incontinence may also occur in epileptic seizure and in nonepileptic events such as syncope. A pooled analysis by Brigo et al11 determined that urinary incontinence had no value in distinguishing between epilepsy and syncope. Therefore, this physical finding should not incline the clinician to one diagnosis or the other.


Given our patient’s presentation, findings on physical examination, and absence of focal neurologic deficits, she should not undergo neuroimaging for syncope evaluation. The more likely cause of her syncope is orthostatic intolerance (orthostatic hypotension or vasovagal syncope) in the setting of intravascular volume depletion, likely secondary to diuretic use. Obtaining orthostatic vital signs is mandatory, and this confirms the diagnosis.

References
  1. Shen WK, Sheldon RS, Benditt DG, et al. 2017 ACC/AHA/HRS guideline for the evaluation and management of patients with syncope: a report of the American College of Cardiology/American Heart Association Task Force on Clinical Practice Guidelines and the Heart Rhythm Society. J Am Coll Cardiol 2017; 70(5):e39–e110. doi:10.1016/j.jacc.2017.03.003
  2. Task Force for the Diagnosis and Management of Syncope; European Society of Cardiology (ESC); European Heart Rhythm Association (EHRA); Heart Failure Association (HFA); Heart Rhythm Society (HRS), Moya A, Sutton R, Ammirati F, et al. Guidelines for the diagnosis and management of syncope (version 2009). Eur Heart J 2009; 30(21):2631–2671. doi:10.1093/eurheartj/ehp298
  3. Mehlsen J, Kaijer MN, Mehlsen AB. Autonomic and electrocardiographic changes in cardioinhibitory syncope. Europace 2008; 10(1):91–95. doi:10.1093/europace/eum237
  4. Goyal N, Donnino MW, Vachhani R, Bajwa R, Ahmad T, Otero R. The utility of head computed tomography in the emergency department evaluation of syncope. Intern Emerg Med 2006; 1(2):148–150. pmid:17111790
  5. Kapoor WN, Karpf M, Wieand S, Peterson JR, Levey GS. A prospective evaluation and follow-up of patients with syncope. N Engl J Med 1983; 309(4):197–204. doi:10.1056/NEJM198307283090401
  6. Sheldon R. How to differentiate syncope from seizure. Cardiol Clin 2015; 33(3):377–385. doi:10.1016/j.ccl.2015.04.006
  7. Raj V, Rowe AA, Fleisch SB, Paranjape SY, Arain AM, Nicolson SE. Psychogenic pseudosyncope: diagnosis and management. Auton Neurosci 2014; 184:66–72. doi:10.1016/j.autneu.2014.05.003
  8. Mecarelli O, Pulitano P, Vicenzini E, Vanacore N, Accornero N, De Marinis M. Observations on EEG patterns in neurally-mediated syncope: an inspective and quantitative study. Neurophysiol Clin 2004; 34(5):203–207. doi:10.1016/j.neucli.2004.09.004
  9. Johnson PC, Ammar H, Zohdy W, Fouda R, Govindu R. Yield of diagnostic tests and its impact on cost in adult patients with syncope presenting to a community hospital. South Med J 2014; 107(11):707–714. doi:10.14423/SMJ.0000000000000184
  10. Sclafani JJ, My J, Zacher LL, Eckart RE. Intensive education on evidence-based evaluation of syncope increases sudden death risk stratification but fails to reduce use of neuroimaging. Arch Intern Med 2010; 170(13):1150–1154. doi:10.1001/archinternmed.2010.205
  11. Brigo F, Nardone R Ausserer H, et al. The diagnostic value of urinary incontinence in the differential diagnosis of seizures. Seizure 2013; 22(2):85–90. doi:10.1016/j.seizure.2012.10.011
Article PDF
Author and Disclosure Information

Erika Hutt-Centeno, MD
Department of Internal Medicine, Cleveland Clinic; Clinical Instructor, Cleveland Clinic Lerner College of Medicine of Case Western Reserve University, Cleveland, OH

Robert Wilson, DO
Neuromuscular Center, Cleveland Clinic; Clinical Assistant Professor, Cleveland Clinic Lerner College of Medicine of Case Western Reserve University, Cleveland, OH

Kenneth A. Mayuga, MD, FHRS, FACC, FACP
Associate Director, Syncope Center; Section of Cardiac Electrophysiology and Pacing, Department of Cardiovascular Medicine, Cleveland Clinic; Assistant Professor of Medicine, Cleveland Clinic Lerner College of Medicine of Case Western Reserve University, Cleveland, OH

Address: Erika Hutt-Centeno, MD, Department of Internal Medicine, G10, Cleveland Clinic, 9500 Euclid Avenue, Cleveland, OH 44195; huttcee@ccf.org

Issue
Cleveland Clinic Journal of Medicine - 86(4)
Publications
Topics
Page Number
240-242
Legacy Keywords
syncope, fainting, lightheadedness, hypotension, arrhythmia, testing, echocardiography, electroencephalography, EEG, tilt table, Erika Hutt-Centeno, Robert Wilson, Kenneth Mayuga
Sections
Author and Disclosure Information

Erika Hutt-Centeno, MD
Department of Internal Medicine, Cleveland Clinic; Clinical Instructor, Cleveland Clinic Lerner College of Medicine of Case Western Reserve University, Cleveland, OH

Robert Wilson, DO
Neuromuscular Center, Cleveland Clinic; Clinical Assistant Professor, Cleveland Clinic Lerner College of Medicine of Case Western Reserve University, Cleveland, OH

Kenneth A. Mayuga, MD, FHRS, FACC, FACP
Associate Director, Syncope Center; Section of Cardiac Electrophysiology and Pacing, Department of Cardiovascular Medicine, Cleveland Clinic; Assistant Professor of Medicine, Cleveland Clinic Lerner College of Medicine of Case Western Reserve University, Cleveland, OH

Address: Erika Hutt-Centeno, MD, Department of Internal Medicine, G10, Cleveland Clinic, 9500 Euclid Avenue, Cleveland, OH 44195; huttcee@ccf.org

Author and Disclosure Information

Erika Hutt-Centeno, MD
Department of Internal Medicine, Cleveland Clinic; Clinical Instructor, Cleveland Clinic Lerner College of Medicine of Case Western Reserve University, Cleveland, OH

Robert Wilson, DO
Neuromuscular Center, Cleveland Clinic; Clinical Assistant Professor, Cleveland Clinic Lerner College of Medicine of Case Western Reserve University, Cleveland, OH

Kenneth A. Mayuga, MD, FHRS, FACC, FACP
Associate Director, Syncope Center; Section of Cardiac Electrophysiology and Pacing, Department of Cardiovascular Medicine, Cleveland Clinic; Assistant Professor of Medicine, Cleveland Clinic Lerner College of Medicine of Case Western Reserve University, Cleveland, OH

Address: Erika Hutt-Centeno, MD, Department of Internal Medicine, G10, Cleveland Clinic, 9500 Euclid Avenue, Cleveland, OH 44195; huttcee@ccf.org

Article PDF
Article PDF
Related Articles

A 40-year-old woman with a history of hypertension, who was recently started on a diuretic, presents to the emergency department after a witnessed syncopal event. She reports a prodrome of lightheadedness, nausea, and darkening of her vision that occurred a few seconds after standing, followed by loss of consciousness. She had a complete, spontaneous recovery after 10 seconds, but upon arousal she noticed she had lost bladder control.

Her blood pressure is 120/80 mm Hg supine, 110/70 mm Hg sitting, and 90/60 mm Hg standing. She has no focal neurologic deficits. The cardiac examination is normal, without murmurs, and electrocardiography shows sinus tachycardia (heart rate 110 bpm) without other abnormalities. Results of laboratory testing are unremarkable.

Should you order neuroimaging to evaluate for syncope?

DEFINITIONS, CLASSIFICATIONS

Syncope is an abrupt loss of consciousness due to transient global cerebral hypoperfusion, with a concomitant loss of postural tone and rapid, spontaneous recovery.1 Recovery from syncope is characterized by immediate restoration of orientation and normal behavior, although the period after recovery may be accompanied by fatigue.2

The European Society of Cardiology2 has classified syncope into 3 main categories: reflex (neurally mediated) syncope, syncope due to orthostatic hypotension, and cardiac syncope. Determining the cause is critical, as this determines the prognosis.

KEYS TO THE EVALUATION

According to the 2017 American College of Cardiology/American Heart Association (ACC/AHA) and the 2009 European Society of Cardiology guidelines, the evaluation of syncope should include a thorough history, taken from the patient and witnesses, and a complete physical examination. This can identify the cause of syncope in up to 50% of cases and differentiate between cardiac and noncardiac causes. Features that point to cardiac syncope include age older than 60, male sex, known heart disease, brief prodrome, syncope during exertion or when supine, first syncopal event, family history of sudden cardiac death, and abnormal physical examination.1

Features that suggest noncardiac syncope are young age; syncope only when standing; recurrent syncope; a prodrome of nausea, vomiting, and a warm sensation; and triggers such as dehydration, pain, distressful stimulus, cough, laugh micturition, defecation, and swallowing.1

Electrocardiography should follow the history and physical examination. When done at presentation, electrocardiography is diagnostic in only about 5% of cases. However, given the importance of the diagnosis, it remains an essential part of the initial evaluation of syncope.3

If a clear cause of syncope is identified at this point, no further workup is needed, and the cause of syncope should be addressed.1 If the cause is still unclear, the ACC/AHA guidelines recommend further evaluation based on the clinical presentation and risk stratification.

 

 

WHEN TO PURSUE ADDITIONAL TESTING

Routine use of additional testing is costly; tests should be ordered on the basis of their potential diagnostic and prognostic value. Additional evaluation should follow a stepwise approach and can include targeted blood work, autonomic nerve evaluation, tilt-table testing, transthoracic echocardiography, stress testing, electrocardiographic monitoring, and electrophysiologic testing.1

Table 1. Criteria for high cardiac risk in syncope
If the initial evaluation indicates cardiac syncope (Table 1), evaluation with echocardiography has a class IIa recommendation (considered reasonable).1,2

Syncope is rarely a manifestation of neurologic disease, yet 11% to 58% of patients with a first episode of uncomplicated syncope undergo extensive neuroimaging with magnetic resonance imaging, computed tomography, electroencephalography (EEG), and carotid ultrasonography.4 Evidence suggests that routine neurologic testing is of limited value given its low diagnostic yield and high cost.

Epilepsy is the most common neurologic cause of loss of consciousness but is estimated to account for less than 5% of patients with syncope.5 A thorough and thoughtful neurologic history and examination is often enough to distinguish between syncope, convulsive syncope, epileptic convulsions, and pseudosyncope.

In syncope, the loss of consciousness usually occurs 30 seconds to several minutes after standing. It presents with or without a prodrome (warmth, palpitations, and diaphoresis) and can be relieved with supine positioning. True loss of consciousness usually lasts less than a minute and is accompanied by loss of postural tone, with little or no fatigue in the recovery period.6

Conversely, in convulsive syncope, the prodrome can include pallor and diaphoresis. Loss of consciousness lasts about 30 seconds but is accompanied by fixed gaze, upward eye deviation, nuchal rigidity, tonic spasms, myoclonic jerks, tonic-clonic convulsions, and oral automatisms.6

Pseudosyncope is characterized by a prodrome of lightheadedness, shortness of breath, chest pain, and tingling sensations, followed by episodes of apparent loss of consciousness that last longer than several minutes and occur multiple times a day. During these episodes, patients purposefully try to avoid trauma when they lose consciousness, and almost always keep their eyes closed, in contrast to syncopal episodes, when the eyes are open and glassy.7

ROLE OF ELECTROENCEPHALOGRAPHY

If the diagnosis remains unclear after the history and neurologic examination, EEG is recommended (class IIa, ie, reasonable, can be useful) during tilt-table testing, as it can help differentiate syncope, pseudosyncope, and epilepsy.1

In an epileptic convulsion, EEG shows epileptiform discharges, whereas in syncope, it shows diffuse brainwave slowing with delta waves and a flatline pattern. In pseudosyncope and psychogenic nonepileptic seizures, EEG shows normal activity.8

Routine EEG is not recommended if there are no specific neurologic signs of epilepsy or if the history and neurologic examination indicate syncope or pseudosyncope.1

Structural brain disease does not typically present with transient global cerebral hypoperfusion resulting in syncope, so magnetic resonance imaging and computed tomography have a low diagnostic yield. Studies have revealed that for the 11% to 58% of patients who undergo neuroimaging, it establishes a diagnosis in only 0.2% to 1%.9 For this reason and in view of their high cost, these imaging tests should not be routinely ordered in the evaluation of syncope.4,10 Similarly, carotid artery imaging should not be routinely ordered if there is no focal neurologic finding suggesting unilateral ischemia.10

CASE CONTINUED

In our 40-year-old patient, the history suggests dehydration, as she recently started taking a diuretic. Thus, laboratory testing is reasonable.

Loss of bladder control is often interpreted as a red flag for neurologic disease, but syncope can often present with urinary incontinence. Urinary incontinence may also occur in epileptic seizure and in nonepileptic events such as syncope. A pooled analysis by Brigo et al11 determined that urinary incontinence had no value in distinguishing between epilepsy and syncope. Therefore, this physical finding should not incline the clinician to one diagnosis or the other.


Given our patient’s presentation, findings on physical examination, and absence of focal neurologic deficits, she should not undergo neuroimaging for syncope evaluation. The more likely cause of her syncope is orthostatic intolerance (orthostatic hypotension or vasovagal syncope) in the setting of intravascular volume depletion, likely secondary to diuretic use. Obtaining orthostatic vital signs is mandatory, and this confirms the diagnosis.

A 40-year-old woman with a history of hypertension, who was recently started on a diuretic, presents to the emergency department after a witnessed syncopal event. She reports a prodrome of lightheadedness, nausea, and darkening of her vision that occurred a few seconds after standing, followed by loss of consciousness. She had a complete, spontaneous recovery after 10 seconds, but upon arousal she noticed she had lost bladder control.

Her blood pressure is 120/80 mm Hg supine, 110/70 mm Hg sitting, and 90/60 mm Hg standing. She has no focal neurologic deficits. The cardiac examination is normal, without murmurs, and electrocardiography shows sinus tachycardia (heart rate 110 bpm) without other abnormalities. Results of laboratory testing are unremarkable.

Should you order neuroimaging to evaluate for syncope?

DEFINITIONS, CLASSIFICATIONS

Syncope is an abrupt loss of consciousness due to transient global cerebral hypoperfusion, with a concomitant loss of postural tone and rapid, spontaneous recovery.1 Recovery from syncope is characterized by immediate restoration of orientation and normal behavior, although the period after recovery may be accompanied by fatigue.2

The European Society of Cardiology2 has classified syncope into 3 main categories: reflex (neurally mediated) syncope, syncope due to orthostatic hypotension, and cardiac syncope. Determining the cause is critical, as this determines the prognosis.

KEYS TO THE EVALUATION

According to the 2017 American College of Cardiology/American Heart Association (ACC/AHA) and the 2009 European Society of Cardiology guidelines, the evaluation of syncope should include a thorough history, taken from the patient and witnesses, and a complete physical examination. This can identify the cause of syncope in up to 50% of cases and differentiate between cardiac and noncardiac causes. Features that point to cardiac syncope include age older than 60, male sex, known heart disease, brief prodrome, syncope during exertion or when supine, first syncopal event, family history of sudden cardiac death, and abnormal physical examination.1

Features that suggest noncardiac syncope are young age; syncope only when standing; recurrent syncope; a prodrome of nausea, vomiting, and a warm sensation; and triggers such as dehydration, pain, distressful stimulus, cough, laugh micturition, defecation, and swallowing.1

Electrocardiography should follow the history and physical examination. When done at presentation, electrocardiography is diagnostic in only about 5% of cases. However, given the importance of the diagnosis, it remains an essential part of the initial evaluation of syncope.3

If a clear cause of syncope is identified at this point, no further workup is needed, and the cause of syncope should be addressed.1 If the cause is still unclear, the ACC/AHA guidelines recommend further evaluation based on the clinical presentation and risk stratification.

 

 

WHEN TO PURSUE ADDITIONAL TESTING

Routine use of additional testing is costly; tests should be ordered on the basis of their potential diagnostic and prognostic value. Additional evaluation should follow a stepwise approach and can include targeted blood work, autonomic nerve evaluation, tilt-table testing, transthoracic echocardiography, stress testing, electrocardiographic monitoring, and electrophysiologic testing.1

Table 1. Criteria for high cardiac risk in syncope
If the initial evaluation indicates cardiac syncope (Table 1), evaluation with echocardiography has a class IIa recommendation (considered reasonable).1,2

Syncope is rarely a manifestation of neurologic disease, yet 11% to 58% of patients with a first episode of uncomplicated syncope undergo extensive neuroimaging with magnetic resonance imaging, computed tomography, electroencephalography (EEG), and carotid ultrasonography.4 Evidence suggests that routine neurologic testing is of limited value given its low diagnostic yield and high cost.

Epilepsy is the most common neurologic cause of loss of consciousness but is estimated to account for less than 5% of patients with syncope.5 A thorough and thoughtful neurologic history and examination is often enough to distinguish between syncope, convulsive syncope, epileptic convulsions, and pseudosyncope.

In syncope, the loss of consciousness usually occurs 30 seconds to several minutes after standing. It presents with or without a prodrome (warmth, palpitations, and diaphoresis) and can be relieved with supine positioning. True loss of consciousness usually lasts less than a minute and is accompanied by loss of postural tone, with little or no fatigue in the recovery period.6

Conversely, in convulsive syncope, the prodrome can include pallor and diaphoresis. Loss of consciousness lasts about 30 seconds but is accompanied by fixed gaze, upward eye deviation, nuchal rigidity, tonic spasms, myoclonic jerks, tonic-clonic convulsions, and oral automatisms.6

Pseudosyncope is characterized by a prodrome of lightheadedness, shortness of breath, chest pain, and tingling sensations, followed by episodes of apparent loss of consciousness that last longer than several minutes and occur multiple times a day. During these episodes, patients purposefully try to avoid trauma when they lose consciousness, and almost always keep their eyes closed, in contrast to syncopal episodes, when the eyes are open and glassy.7

ROLE OF ELECTROENCEPHALOGRAPHY

If the diagnosis remains unclear after the history and neurologic examination, EEG is recommended (class IIa, ie, reasonable, can be useful) during tilt-table testing, as it can help differentiate syncope, pseudosyncope, and epilepsy.1

In an epileptic convulsion, EEG shows epileptiform discharges, whereas in syncope, it shows diffuse brainwave slowing with delta waves and a flatline pattern. In pseudosyncope and psychogenic nonepileptic seizures, EEG shows normal activity.8

Routine EEG is not recommended if there are no specific neurologic signs of epilepsy or if the history and neurologic examination indicate syncope or pseudosyncope.1

Structural brain disease does not typically present with transient global cerebral hypoperfusion resulting in syncope, so magnetic resonance imaging and computed tomography have a low diagnostic yield. Studies have revealed that for the 11% to 58% of patients who undergo neuroimaging, it establishes a diagnosis in only 0.2% to 1%.9 For this reason and in view of their high cost, these imaging tests should not be routinely ordered in the evaluation of syncope.4,10 Similarly, carotid artery imaging should not be routinely ordered if there is no focal neurologic finding suggesting unilateral ischemia.10

CASE CONTINUED

In our 40-year-old patient, the history suggests dehydration, as she recently started taking a diuretic. Thus, laboratory testing is reasonable.

Loss of bladder control is often interpreted as a red flag for neurologic disease, but syncope can often present with urinary incontinence. Urinary incontinence may also occur in epileptic seizure and in nonepileptic events such as syncope. A pooled analysis by Brigo et al11 determined that urinary incontinence had no value in distinguishing between epilepsy and syncope. Therefore, this physical finding should not incline the clinician to one diagnosis or the other.


Given our patient’s presentation, findings on physical examination, and absence of focal neurologic deficits, she should not undergo neuroimaging for syncope evaluation. The more likely cause of her syncope is orthostatic intolerance (orthostatic hypotension or vasovagal syncope) in the setting of intravascular volume depletion, likely secondary to diuretic use. Obtaining orthostatic vital signs is mandatory, and this confirms the diagnosis.

References
  1. Shen WK, Sheldon RS, Benditt DG, et al. 2017 ACC/AHA/HRS guideline for the evaluation and management of patients with syncope: a report of the American College of Cardiology/American Heart Association Task Force on Clinical Practice Guidelines and the Heart Rhythm Society. J Am Coll Cardiol 2017; 70(5):e39–e110. doi:10.1016/j.jacc.2017.03.003
  2. Task Force for the Diagnosis and Management of Syncope; European Society of Cardiology (ESC); European Heart Rhythm Association (EHRA); Heart Failure Association (HFA); Heart Rhythm Society (HRS), Moya A, Sutton R, Ammirati F, et al. Guidelines for the diagnosis and management of syncope (version 2009). Eur Heart J 2009; 30(21):2631–2671. doi:10.1093/eurheartj/ehp298
  3. Mehlsen J, Kaijer MN, Mehlsen AB. Autonomic and electrocardiographic changes in cardioinhibitory syncope. Europace 2008; 10(1):91–95. doi:10.1093/europace/eum237
  4. Goyal N, Donnino MW, Vachhani R, Bajwa R, Ahmad T, Otero R. The utility of head computed tomography in the emergency department evaluation of syncope. Intern Emerg Med 2006; 1(2):148–150. pmid:17111790
  5. Kapoor WN, Karpf M, Wieand S, Peterson JR, Levey GS. A prospective evaluation and follow-up of patients with syncope. N Engl J Med 1983; 309(4):197–204. doi:10.1056/NEJM198307283090401
  6. Sheldon R. How to differentiate syncope from seizure. Cardiol Clin 2015; 33(3):377–385. doi:10.1016/j.ccl.2015.04.006
  7. Raj V, Rowe AA, Fleisch SB, Paranjape SY, Arain AM, Nicolson SE. Psychogenic pseudosyncope: diagnosis and management. Auton Neurosci 2014; 184:66–72. doi:10.1016/j.autneu.2014.05.003
  8. Mecarelli O, Pulitano P, Vicenzini E, Vanacore N, Accornero N, De Marinis M. Observations on EEG patterns in neurally-mediated syncope: an inspective and quantitative study. Neurophysiol Clin 2004; 34(5):203–207. doi:10.1016/j.neucli.2004.09.004
  9. Johnson PC, Ammar H, Zohdy W, Fouda R, Govindu R. Yield of diagnostic tests and its impact on cost in adult patients with syncope presenting to a community hospital. South Med J 2014; 107(11):707–714. doi:10.14423/SMJ.0000000000000184
  10. Sclafani JJ, My J, Zacher LL, Eckart RE. Intensive education on evidence-based evaluation of syncope increases sudden death risk stratification but fails to reduce use of neuroimaging. Arch Intern Med 2010; 170(13):1150–1154. doi:10.1001/archinternmed.2010.205
  11. Brigo F, Nardone R Ausserer H, et al. The diagnostic value of urinary incontinence in the differential diagnosis of seizures. Seizure 2013; 22(2):85–90. doi:10.1016/j.seizure.2012.10.011
References
  1. Shen WK, Sheldon RS, Benditt DG, et al. 2017 ACC/AHA/HRS guideline for the evaluation and management of patients with syncope: a report of the American College of Cardiology/American Heart Association Task Force on Clinical Practice Guidelines and the Heart Rhythm Society. J Am Coll Cardiol 2017; 70(5):e39–e110. doi:10.1016/j.jacc.2017.03.003
  2. Task Force for the Diagnosis and Management of Syncope; European Society of Cardiology (ESC); European Heart Rhythm Association (EHRA); Heart Failure Association (HFA); Heart Rhythm Society (HRS), Moya A, Sutton R, Ammirati F, et al. Guidelines for the diagnosis and management of syncope (version 2009). Eur Heart J 2009; 30(21):2631–2671. doi:10.1093/eurheartj/ehp298
  3. Mehlsen J, Kaijer MN, Mehlsen AB. Autonomic and electrocardiographic changes in cardioinhibitory syncope. Europace 2008; 10(1):91–95. doi:10.1093/europace/eum237
  4. Goyal N, Donnino MW, Vachhani R, Bajwa R, Ahmad T, Otero R. The utility of head computed tomography in the emergency department evaluation of syncope. Intern Emerg Med 2006; 1(2):148–150. pmid:17111790
  5. Kapoor WN, Karpf M, Wieand S, Peterson JR, Levey GS. A prospective evaluation and follow-up of patients with syncope. N Engl J Med 1983; 309(4):197–204. doi:10.1056/NEJM198307283090401
  6. Sheldon R. How to differentiate syncope from seizure. Cardiol Clin 2015; 33(3):377–385. doi:10.1016/j.ccl.2015.04.006
  7. Raj V, Rowe AA, Fleisch SB, Paranjape SY, Arain AM, Nicolson SE. Psychogenic pseudosyncope: diagnosis and management. Auton Neurosci 2014; 184:66–72. doi:10.1016/j.autneu.2014.05.003
  8. Mecarelli O, Pulitano P, Vicenzini E, Vanacore N, Accornero N, De Marinis M. Observations on EEG patterns in neurally-mediated syncope: an inspective and quantitative study. Neurophysiol Clin 2004; 34(5):203–207. doi:10.1016/j.neucli.2004.09.004
  9. Johnson PC, Ammar H, Zohdy W, Fouda R, Govindu R. Yield of diagnostic tests and its impact on cost in adult patients with syncope presenting to a community hospital. South Med J 2014; 107(11):707–714. doi:10.14423/SMJ.0000000000000184
  10. Sclafani JJ, My J, Zacher LL, Eckart RE. Intensive education on evidence-based evaluation of syncope increases sudden death risk stratification but fails to reduce use of neuroimaging. Arch Intern Med 2010; 170(13):1150–1154. doi:10.1001/archinternmed.2010.205
  11. Brigo F, Nardone R Ausserer H, et al. The diagnostic value of urinary incontinence in the differential diagnosis of seizures. Seizure 2013; 22(2):85–90. doi:10.1016/j.seizure.2012.10.011
Issue
Cleveland Clinic Journal of Medicine - 86(4)
Issue
Cleveland Clinic Journal of Medicine - 86(4)
Page Number
240-242
Page Number
240-242
Publications
Publications
Topics
Article Type
Display Headline
Is neuroimaging necessary to evaluate syncope?
Display Headline
Is neuroimaging necessary to evaluate syncope?
Legacy Keywords
syncope, fainting, lightheadedness, hypotension, arrhythmia, testing, echocardiography, electroencephalography, EEG, tilt table, Erika Hutt-Centeno, Robert Wilson, Kenneth Mayuga
Legacy Keywords
syncope, fainting, lightheadedness, hypotension, arrhythmia, testing, echocardiography, electroencephalography, EEG, tilt table, Erika Hutt-Centeno, Robert Wilson, Kenneth Mayuga
Sections
Disallow All Ads
Content Gating
No Gating (article Unlocked/Free)
Alternative CME
Disqus Comments
Default
Gate On Date
Mon, 03/25/2019 - 14:30
Un-Gate On Date
Mon, 03/25/2019 - 14:30
Use ProPublica
CFC Schedule Remove Status
Mon, 03/25/2019 - 14:30
Hide sidebar & use full width
render the right sidebar.
Article PDF Media

How long should we follow simple ovarian cysts with pelvic ultrasonography?

Article Type
Changed
Wed, 10/10/2018 - 10:17
Display Headline
How long should we follow simple ovarian cysts with pelvic ultrasonography?

A 54-year-old postmenopausal woman presents with a 3-day history of left lower quadrant pain. Abdominal and pelvic computed tomography confirm the diagnosis of acute diverticulitis, and a left ovarian cyst is incidentally noted. Her abdominal discomfort resolves with antibiotics.

Transvaginal ultrasonography confirms the presence of a 4.5-cm simple left ovarian cyst. The radiologist recommends follow-up ultrasonography in 3 months “if clinically indicated.” The patient feels well and is anxious about having additional testing. What do you recommend?

HOW USEFUL IS ULTRASONOGRAPHY FOR OVARIAN CYSTS?

Ovarian cysts are common and may affect up to 20% of women at some time during their life.1 In a prospective study of almost 40,000 women enrolled in an ovarian cancer screening program, the prevalence of ovarian cysts was 15.3% in premenopausal women and 8.2% in postmenopausal women.2

Pelvic ultrasonography is the most effective way to evaluate incidentally noted cysts, and the transvaginal approach is preferred.3 The International Ovarian Tumor Analysis group has outlined morphologic features, referred to as “simple rules,” for predicting if a cyst is malignant or benign.4 In a prospective validation study, these simple rules were applied in 76% of cases, with a sensitivity of 95% and a specificity of 91%.4 However, it should be noted that these rules apply to examinations done by experienced gynecologic ultrasonographers, as accuracy of ultrasonography is both machine- and operator-dependent.

WHAT IS THE MALIGNANCY POTENTIAL OF A SIMPLE OVARIAN CYST?

A simple ovarian cyst is defined as an anechoic round or oval lesion, different from a unilocular cyst, which may contain septations, solid wall irregularities, or internal echoes.5 Overall, simple ovarian cysts have a very low likelihood of malignancy. In the large, multi-site Prostate, Lung, Colorectal, and Ovarian Cancer Screening Trial, simple cysts were observed in 14% of postmenopausal women,6 but no cyst was associated with the development of ovarian cancer over 4 years of follow-up.

HOW OFTEN SHOULD IMAGING BE REPEATED?

In premenopausal women, most simple (thin-walled) ovarian cysts less than 5 cm in maximum diameter resolve in 2 to 3 menstrual cycles and do not require further intervention.3 Larger cysts (5–7 cm in diameter) should be followed with ultrasonography yearly. Cysts larger than 7 cm require advanced imaging or surgical intervention, and the patient should be referred to a gynecologist.3

In postmenopausal women, serum markers are combined with ultrasonography results to determine the risk of malignancy. Markers studied include cancer antigen 125 (CA-125), human epididymis protein 4, lactate dehydrogenase, alpha fetoprotein, and beta human chorionic gonadotropin (beta hCG).7

CA-125, the most studied marker, is elevated in more than 90% of advanced-stage ovarian cancers, but in only 50% of patients wth early-stage cancer.1,8 However, CA-125 may be elevated in a variety of other settings, including benign gynecologic disorders (pelvic infection, fibroids, endometriosis, adenomyosis) and nongynecologic disorders (liver disease, pancreatitis, and diverticulitis). Thus, it is unreliable for distinguishing benign from malignant ovarian masses in premenopausal women.1,3

Current guidelines recommend routine measurement of CA-125 in the initial evaluation of all postmenopausal women with an ovarian mass.7,8

Using a cutoff of 30 IU/mL, CA-125 has a sensitivity of 81% and a specificity of 75% for ovarian malignancy. However, serial measurements may be more useful for assessing ovarian cancer risk, especially in the setting of rapidly rising values.1,3

The Risk for Malignancy Index (RMI), which categorizes a cyst’s risk for malignancy, can be calculated based on the patient’s menopausal status, ultrasonographic characteristics (1 point each for multilocular cyst, solid area, metastasis, ascites, and bilateral lesions), and serum CA-125 level. The RMI has a sensitivity of 78% and a specificity of 87% for predicting ovarian cancer.8

Postmenopausal women with an asymptomatic small cyst (< 5 cm), a normal CA-125 level, and an RMI < 200 can be followed conservatively, with repeat ultrasonography in 4 to 6 months. At that time, if the cyst has not grown and the CA-125 level is normal, expectant management can continue, with reassessment in 4 to 6 months. If imaging remains unchanged and the CA-125 is persistently normal, the patient may be discharged from follow-up.8

If at any time during the evaluation the calculated RMI is greater than 200, there is an increased risk for malignancy, and the patient should be referred to a gynecologic oncologist for advanced imaging.

An algorithm from the Royal College of Obstetricians and Gynaecologists for managing ovarian cysts in postmenopausal women is available at www.rcog.org.uk/globalassets/documents/guidelines/green-top-guidelines/gtg_34.pdf.8

 

 

CURRENT GUIDELINES ON REPEAT IMAGING

The American College of Radiology (ACR) has created a “Choosing Wisely” guideline to clarify when repeat imaging for ovarian cysts is indicated, to reduce both patient anxiety and healthcare costs.9 These guidelines highlight the distress women may experience from repeat testing due to concerns about cancer.

The guidelines also note that testing is often done during varying times of the menstrual cycle, thereby detecting new cysts, as opposed to monitoring previously detected cysts. Repeat ultrasonography may lead to surgical interventions that are not evidence-based, such as cystectomy or oophorectomy, in patients without radiologic features of malignancy or associated pelvic pain. And while ultrasonography is less expensive than other imaging tests, unnecessary imaging can mean additional costs to the patient, such as copayments, and possibly large payments for patients without insurance.

The American College of Obstetricians and Gynecologists (ACOG) and the ACR guidelines recommend against unnecessary repeat imaging for ovarian cysts.7,10 The ACOG Practice Bulletin on the Evaluation and Management of Adnexal Masses states, “Simple cysts up to 10 cm in diameter on transvaginal ultrasonography performed by experienced ultrasonographers are likely benign and may be safely monitored using repeat imaging without surgical intervention, even in postmenopausal patients.”7

The ideal frequency for repeat testing is yet to be determined. In postmenopausal women with a simple cyst smaller than 5 cm, ACOG guidelines recommend an interval of 4 to 6 months for initial repeat imaging. ACR guidelines recommend no follow-up imaging for simple cysts smaller than 5 cm detected by high-quality ultrasonography in asymptomatic women of reproductive age or for simple cysts smaller than 1 cm in postmenopausal women.10

THE CLINICAL BOTTOM LINE

Simple ovarian cysts can develop as part of the normal menstrual cycle, and although they are more common in premenopausal women, they have been detected in 1 out of 5 postmenopausal women.9 Simple ovarian cysts are typically not cancerous in women of any age. Therefore, most simple ovarian cysts in asymp­tomatic women either require no follow-up imaging or can be safely monitored with limited repeat ultrasonography for a defined length of time.

Our 54-year-old postmenopausal patient has a simple cyst smaller than 5 cm. Based on current guidelines, the CA-125 level should be measured, with subsequent calculation of the RMI. Assuming a normal CA-125 and RMI, she should be reassured that the risk of progression to malignancy is extremely low. Repeating ultrasonography 4 to 6 months after the initial imaging is warranted. At that time, if no change in cyst size or composition is detected, ultrasonography can be repeated at 1 year after initial detection. After that, assuming no changes of the cyst on repeat imaging, the patient does not require additional follow-up.

References
  1. van Nagell JR Jr, Miiler RW. Evaluation and management of ultrasonographically detected ovarian tumors in asymptomatic women. Obstet Gynecol 2016; 127(5):848–858. doi:10.1097/AOG.0000000000001384
  2. Pavlik EJ, Ueland FR, Miller RW, et al. Frequency and disposition of ovarian abnormalities followed with serial transvaginal ultrasonography. Obstet Gynecol 2013; 122(2 pt 1):210–217. doi:10.1097/AOG.0b013e318298def5
  3. Royal College of Obstetricians and Gynaecologists. Management of suspected ovarian masses in premenopausal women. Green-top guideline 2011; 62:1–14. www.rcog.org.uk/globalassets/documents/guidelines/gtg_62.pdf. Accessed August 16, 2018.
  4. Timmerman D, Testa AC, Bourne T, et al. Simple ultrasound-based rules for the diagnosis of ovarian cancer. Ultrasound Obstet Gynecol 2008; 31(6):681–690. doi:10.1002/uog.5365
  5. Glanc P, Benacerraf B, Bourne T, et al. First international consensus report on adnexal masses: management recommendations. J Ultrasound Med 2017; 36(5):849–863. doi:10.1002/jum.14197
  6. Greenlee RT, Kessel B, Williams CR, et al. Prevalence, incidence, and natural history of simple ovarian cysts among women > 55 years old in a large cancer screening trial. Am J Obstet Gynecol 2010; 202(4):373.e1–e9. doi:10.1016/j.ajog.2009.11.029
  7. American College of Obstetricians and Gynecologists’ Committee on Practice Bulletins—Gynecology. Practice Bulletin No. 174: Evaluation and Management of Adnexal Masses. Obstet Gynecol 2016; 128(5):e210-226. doi:10.1097/AOG.0000000000001768
  8. Royal College of Obstetricians and Gynaecologists. The management of ovarian cysts in postmenopausal women. Green-top guideline 2016; 34:1–31. www.rcog.org.uk/globalassets/documents/guidelines/green-top-guidelines/gtg_34.pdf. Accessed August 16, 2018.
  9. American College of Radiology Choosing Wisely. Imaging tests for ovarian cysts. When you need an ultrasound—and when you don’t. www.choosingwisely.org/wp-content/uploads/2012/09/ChoosingWiselyOvarianCystsACR-ER_Update.pdf. Accessed August 16, 2018.
  10. Levine D, Brown DL, Andreotti RF, et al. Management of asymptomatic ovarian and other adnexal cysts imaged at US: Society of Radiologists in Ultrasound Consensus Conference Statement. Radiology 2010; 256(3):943–954. doi:10.1148/radiol.10100213
Article PDF
Author and Disclosure Information

Megan C. McNamara, MD, MSc
Department of Medicine, Case Western Reserve University, Cleveland, OH; Louis Stokes Cleveland VA Medical Center, Cleveland, OH; Associate Professor of Medicine, Case Western Reserve University School of Medicine, Cleveland, OH

Rachel Brook, MD
Department of Medicine, Iris Cantor UCLA Women’s Health Center, University of California, Los Angeles

Address: Megan C. McNamara, MD, MSc, Louis Stokes Cleveland VA Medical Center, 10701 East Boulevard, Cleveland, OH 44106; megan.mcnamara@va.gov

Issue
Cleveland Clinic Journal of Medicine - 85(10)
Publications
Topics
Page Number
745-747
Legacy Keywords
ovarian cysts, ultrasonography, ultrasound, cancer antigen 125, CA-125, Megan McNamara, Rachel Brook
Sections
Author and Disclosure Information

Megan C. McNamara, MD, MSc
Department of Medicine, Case Western Reserve University, Cleveland, OH; Louis Stokes Cleveland VA Medical Center, Cleveland, OH; Associate Professor of Medicine, Case Western Reserve University School of Medicine, Cleveland, OH

Rachel Brook, MD
Department of Medicine, Iris Cantor UCLA Women’s Health Center, University of California, Los Angeles

Address: Megan C. McNamara, MD, MSc, Louis Stokes Cleveland VA Medical Center, 10701 East Boulevard, Cleveland, OH 44106; megan.mcnamara@va.gov

Author and Disclosure Information

Megan C. McNamara, MD, MSc
Department of Medicine, Case Western Reserve University, Cleveland, OH; Louis Stokes Cleveland VA Medical Center, Cleveland, OH; Associate Professor of Medicine, Case Western Reserve University School of Medicine, Cleveland, OH

Rachel Brook, MD
Department of Medicine, Iris Cantor UCLA Women’s Health Center, University of California, Los Angeles

Address: Megan C. McNamara, MD, MSc, Louis Stokes Cleveland VA Medical Center, 10701 East Boulevard, Cleveland, OH 44106; megan.mcnamara@va.gov

Article PDF
Article PDF
Related Articles

A 54-year-old postmenopausal woman presents with a 3-day history of left lower quadrant pain. Abdominal and pelvic computed tomography confirm the diagnosis of acute diverticulitis, and a left ovarian cyst is incidentally noted. Her abdominal discomfort resolves with antibiotics.

Transvaginal ultrasonography confirms the presence of a 4.5-cm simple left ovarian cyst. The radiologist recommends follow-up ultrasonography in 3 months “if clinically indicated.” The patient feels well and is anxious about having additional testing. What do you recommend?

HOW USEFUL IS ULTRASONOGRAPHY FOR OVARIAN CYSTS?

Ovarian cysts are common and may affect up to 20% of women at some time during their life.1 In a prospective study of almost 40,000 women enrolled in an ovarian cancer screening program, the prevalence of ovarian cysts was 15.3% in premenopausal women and 8.2% in postmenopausal women.2

Pelvic ultrasonography is the most effective way to evaluate incidentally noted cysts, and the transvaginal approach is preferred.3 The International Ovarian Tumor Analysis group has outlined morphologic features, referred to as “simple rules,” for predicting if a cyst is malignant or benign.4 In a prospective validation study, these simple rules were applied in 76% of cases, with a sensitivity of 95% and a specificity of 91%.4 However, it should be noted that these rules apply to examinations done by experienced gynecologic ultrasonographers, as accuracy of ultrasonography is both machine- and operator-dependent.

WHAT IS THE MALIGNANCY POTENTIAL OF A SIMPLE OVARIAN CYST?

A simple ovarian cyst is defined as an anechoic round or oval lesion, different from a unilocular cyst, which may contain septations, solid wall irregularities, or internal echoes.5 Overall, simple ovarian cysts have a very low likelihood of malignancy. In the large, multi-site Prostate, Lung, Colorectal, and Ovarian Cancer Screening Trial, simple cysts were observed in 14% of postmenopausal women,6 but no cyst was associated with the development of ovarian cancer over 4 years of follow-up.

HOW OFTEN SHOULD IMAGING BE REPEATED?

In premenopausal women, most simple (thin-walled) ovarian cysts less than 5 cm in maximum diameter resolve in 2 to 3 menstrual cycles and do not require further intervention.3 Larger cysts (5–7 cm in diameter) should be followed with ultrasonography yearly. Cysts larger than 7 cm require advanced imaging or surgical intervention, and the patient should be referred to a gynecologist.3

In postmenopausal women, serum markers are combined with ultrasonography results to determine the risk of malignancy. Markers studied include cancer antigen 125 (CA-125), human epididymis protein 4, lactate dehydrogenase, alpha fetoprotein, and beta human chorionic gonadotropin (beta hCG).7

CA-125, the most studied marker, is elevated in more than 90% of advanced-stage ovarian cancers, but in only 50% of patients wth early-stage cancer.1,8 However, CA-125 may be elevated in a variety of other settings, including benign gynecologic disorders (pelvic infection, fibroids, endometriosis, adenomyosis) and nongynecologic disorders (liver disease, pancreatitis, and diverticulitis). Thus, it is unreliable for distinguishing benign from malignant ovarian masses in premenopausal women.1,3

Current guidelines recommend routine measurement of CA-125 in the initial evaluation of all postmenopausal women with an ovarian mass.7,8

Using a cutoff of 30 IU/mL, CA-125 has a sensitivity of 81% and a specificity of 75% for ovarian malignancy. However, serial measurements may be more useful for assessing ovarian cancer risk, especially in the setting of rapidly rising values.1,3

The Risk for Malignancy Index (RMI), which categorizes a cyst’s risk for malignancy, can be calculated based on the patient’s menopausal status, ultrasonographic characteristics (1 point each for multilocular cyst, solid area, metastasis, ascites, and bilateral lesions), and serum CA-125 level. The RMI has a sensitivity of 78% and a specificity of 87% for predicting ovarian cancer.8

Postmenopausal women with an asymptomatic small cyst (< 5 cm), a normal CA-125 level, and an RMI < 200 can be followed conservatively, with repeat ultrasonography in 4 to 6 months. At that time, if the cyst has not grown and the CA-125 level is normal, expectant management can continue, with reassessment in 4 to 6 months. If imaging remains unchanged and the CA-125 is persistently normal, the patient may be discharged from follow-up.8

If at any time during the evaluation the calculated RMI is greater than 200, there is an increased risk for malignancy, and the patient should be referred to a gynecologic oncologist for advanced imaging.

An algorithm from the Royal College of Obstetricians and Gynaecologists for managing ovarian cysts in postmenopausal women is available at www.rcog.org.uk/globalassets/documents/guidelines/green-top-guidelines/gtg_34.pdf.8

 

 

CURRENT GUIDELINES ON REPEAT IMAGING

The American College of Radiology (ACR) has created a “Choosing Wisely” guideline to clarify when repeat imaging for ovarian cysts is indicated, to reduce both patient anxiety and healthcare costs.9 These guidelines highlight the distress women may experience from repeat testing due to concerns about cancer.

The guidelines also note that testing is often done during varying times of the menstrual cycle, thereby detecting new cysts, as opposed to monitoring previously detected cysts. Repeat ultrasonography may lead to surgical interventions that are not evidence-based, such as cystectomy or oophorectomy, in patients without radiologic features of malignancy or associated pelvic pain. And while ultrasonography is less expensive than other imaging tests, unnecessary imaging can mean additional costs to the patient, such as copayments, and possibly large payments for patients without insurance.

The American College of Obstetricians and Gynecologists (ACOG) and the ACR guidelines recommend against unnecessary repeat imaging for ovarian cysts.7,10 The ACOG Practice Bulletin on the Evaluation and Management of Adnexal Masses states, “Simple cysts up to 10 cm in diameter on transvaginal ultrasonography performed by experienced ultrasonographers are likely benign and may be safely monitored using repeat imaging without surgical intervention, even in postmenopausal patients.”7

The ideal frequency for repeat testing is yet to be determined. In postmenopausal women with a simple cyst smaller than 5 cm, ACOG guidelines recommend an interval of 4 to 6 months for initial repeat imaging. ACR guidelines recommend no follow-up imaging for simple cysts smaller than 5 cm detected by high-quality ultrasonography in asymptomatic women of reproductive age or for simple cysts smaller than 1 cm in postmenopausal women.10

THE CLINICAL BOTTOM LINE

Simple ovarian cysts can develop as part of the normal menstrual cycle, and although they are more common in premenopausal women, they have been detected in 1 out of 5 postmenopausal women.9 Simple ovarian cysts are typically not cancerous in women of any age. Therefore, most simple ovarian cysts in asymp­tomatic women either require no follow-up imaging or can be safely monitored with limited repeat ultrasonography for a defined length of time.

Our 54-year-old postmenopausal patient has a simple cyst smaller than 5 cm. Based on current guidelines, the CA-125 level should be measured, with subsequent calculation of the RMI. Assuming a normal CA-125 and RMI, she should be reassured that the risk of progression to malignancy is extremely low. Repeating ultrasonography 4 to 6 months after the initial imaging is warranted. At that time, if no change in cyst size or composition is detected, ultrasonography can be repeated at 1 year after initial detection. After that, assuming no changes of the cyst on repeat imaging, the patient does not require additional follow-up.

A 54-year-old postmenopausal woman presents with a 3-day history of left lower quadrant pain. Abdominal and pelvic computed tomography confirm the diagnosis of acute diverticulitis, and a left ovarian cyst is incidentally noted. Her abdominal discomfort resolves with antibiotics.

Transvaginal ultrasonography confirms the presence of a 4.5-cm simple left ovarian cyst. The radiologist recommends follow-up ultrasonography in 3 months “if clinically indicated.” The patient feels well and is anxious about having additional testing. What do you recommend?

HOW USEFUL IS ULTRASONOGRAPHY FOR OVARIAN CYSTS?

Ovarian cysts are common and may affect up to 20% of women at some time during their life.1 In a prospective study of almost 40,000 women enrolled in an ovarian cancer screening program, the prevalence of ovarian cysts was 15.3% in premenopausal women and 8.2% in postmenopausal women.2

Pelvic ultrasonography is the most effective way to evaluate incidentally noted cysts, and the transvaginal approach is preferred.3 The International Ovarian Tumor Analysis group has outlined morphologic features, referred to as “simple rules,” for predicting if a cyst is malignant or benign.4 In a prospective validation study, these simple rules were applied in 76% of cases, with a sensitivity of 95% and a specificity of 91%.4 However, it should be noted that these rules apply to examinations done by experienced gynecologic ultrasonographers, as accuracy of ultrasonography is both machine- and operator-dependent.

WHAT IS THE MALIGNANCY POTENTIAL OF A SIMPLE OVARIAN CYST?

A simple ovarian cyst is defined as an anechoic round or oval lesion, different from a unilocular cyst, which may contain septations, solid wall irregularities, or internal echoes.5 Overall, simple ovarian cysts have a very low likelihood of malignancy. In the large, multi-site Prostate, Lung, Colorectal, and Ovarian Cancer Screening Trial, simple cysts were observed in 14% of postmenopausal women,6 but no cyst was associated with the development of ovarian cancer over 4 years of follow-up.

HOW OFTEN SHOULD IMAGING BE REPEATED?

In premenopausal women, most simple (thin-walled) ovarian cysts less than 5 cm in maximum diameter resolve in 2 to 3 menstrual cycles and do not require further intervention.3 Larger cysts (5–7 cm in diameter) should be followed with ultrasonography yearly. Cysts larger than 7 cm require advanced imaging or surgical intervention, and the patient should be referred to a gynecologist.3

In postmenopausal women, serum markers are combined with ultrasonography results to determine the risk of malignancy. Markers studied include cancer antigen 125 (CA-125), human epididymis protein 4, lactate dehydrogenase, alpha fetoprotein, and beta human chorionic gonadotropin (beta hCG).7

CA-125, the most studied marker, is elevated in more than 90% of advanced-stage ovarian cancers, but in only 50% of patients wth early-stage cancer.1,8 However, CA-125 may be elevated in a variety of other settings, including benign gynecologic disorders (pelvic infection, fibroids, endometriosis, adenomyosis) and nongynecologic disorders (liver disease, pancreatitis, and diverticulitis). Thus, it is unreliable for distinguishing benign from malignant ovarian masses in premenopausal women.1,3

Current guidelines recommend routine measurement of CA-125 in the initial evaluation of all postmenopausal women with an ovarian mass.7,8

Using a cutoff of 30 IU/mL, CA-125 has a sensitivity of 81% and a specificity of 75% for ovarian malignancy. However, serial measurements may be more useful for assessing ovarian cancer risk, especially in the setting of rapidly rising values.1,3

The Risk for Malignancy Index (RMI), which categorizes a cyst’s risk for malignancy, can be calculated based on the patient’s menopausal status, ultrasonographic characteristics (1 point each for multilocular cyst, solid area, metastasis, ascites, and bilateral lesions), and serum CA-125 level. The RMI has a sensitivity of 78% and a specificity of 87% for predicting ovarian cancer.8

Postmenopausal women with an asymptomatic small cyst (< 5 cm), a normal CA-125 level, and an RMI < 200 can be followed conservatively, with repeat ultrasonography in 4 to 6 months. At that time, if the cyst has not grown and the CA-125 level is normal, expectant management can continue, with reassessment in 4 to 6 months. If imaging remains unchanged and the CA-125 is persistently normal, the patient may be discharged from follow-up.8

If at any time during the evaluation the calculated RMI is greater than 200, there is an increased risk for malignancy, and the patient should be referred to a gynecologic oncologist for advanced imaging.

An algorithm from the Royal College of Obstetricians and Gynaecologists for managing ovarian cysts in postmenopausal women is available at www.rcog.org.uk/globalassets/documents/guidelines/green-top-guidelines/gtg_34.pdf.8

 

 

CURRENT GUIDELINES ON REPEAT IMAGING

The American College of Radiology (ACR) has created a “Choosing Wisely” guideline to clarify when repeat imaging for ovarian cysts is indicated, to reduce both patient anxiety and healthcare costs.9 These guidelines highlight the distress women may experience from repeat testing due to concerns about cancer.

The guidelines also note that testing is often done during varying times of the menstrual cycle, thereby detecting new cysts, as opposed to monitoring previously detected cysts. Repeat ultrasonography may lead to surgical interventions that are not evidence-based, such as cystectomy or oophorectomy, in patients without radiologic features of malignancy or associated pelvic pain. And while ultrasonography is less expensive than other imaging tests, unnecessary imaging can mean additional costs to the patient, such as copayments, and possibly large payments for patients without insurance.

The American College of Obstetricians and Gynecologists (ACOG) and the ACR guidelines recommend against unnecessary repeat imaging for ovarian cysts.7,10 The ACOG Practice Bulletin on the Evaluation and Management of Adnexal Masses states, “Simple cysts up to 10 cm in diameter on transvaginal ultrasonography performed by experienced ultrasonographers are likely benign and may be safely monitored using repeat imaging without surgical intervention, even in postmenopausal patients.”7

The ideal frequency for repeat testing is yet to be determined. In postmenopausal women with a simple cyst smaller than 5 cm, ACOG guidelines recommend an interval of 4 to 6 months for initial repeat imaging. ACR guidelines recommend no follow-up imaging for simple cysts smaller than 5 cm detected by high-quality ultrasonography in asymptomatic women of reproductive age or for simple cysts smaller than 1 cm in postmenopausal women.10

THE CLINICAL BOTTOM LINE

Simple ovarian cysts can develop as part of the normal menstrual cycle, and although they are more common in premenopausal women, they have been detected in 1 out of 5 postmenopausal women.9 Simple ovarian cysts are typically not cancerous in women of any age. Therefore, most simple ovarian cysts in asymp­tomatic women either require no follow-up imaging or can be safely monitored with limited repeat ultrasonography for a defined length of time.

Our 54-year-old postmenopausal patient has a simple cyst smaller than 5 cm. Based on current guidelines, the CA-125 level should be measured, with subsequent calculation of the RMI. Assuming a normal CA-125 and RMI, she should be reassured that the risk of progression to malignancy is extremely low. Repeating ultrasonography 4 to 6 months after the initial imaging is warranted. At that time, if no change in cyst size or composition is detected, ultrasonography can be repeated at 1 year after initial detection. After that, assuming no changes of the cyst on repeat imaging, the patient does not require additional follow-up.

References
  1. van Nagell JR Jr, Miiler RW. Evaluation and management of ultrasonographically detected ovarian tumors in asymptomatic women. Obstet Gynecol 2016; 127(5):848–858. doi:10.1097/AOG.0000000000001384
  2. Pavlik EJ, Ueland FR, Miller RW, et al. Frequency and disposition of ovarian abnormalities followed with serial transvaginal ultrasonography. Obstet Gynecol 2013; 122(2 pt 1):210–217. doi:10.1097/AOG.0b013e318298def5
  3. Royal College of Obstetricians and Gynaecologists. Management of suspected ovarian masses in premenopausal women. Green-top guideline 2011; 62:1–14. www.rcog.org.uk/globalassets/documents/guidelines/gtg_62.pdf. Accessed August 16, 2018.
  4. Timmerman D, Testa AC, Bourne T, et al. Simple ultrasound-based rules for the diagnosis of ovarian cancer. Ultrasound Obstet Gynecol 2008; 31(6):681–690. doi:10.1002/uog.5365
  5. Glanc P, Benacerraf B, Bourne T, et al. First international consensus report on adnexal masses: management recommendations. J Ultrasound Med 2017; 36(5):849–863. doi:10.1002/jum.14197
  6. Greenlee RT, Kessel B, Williams CR, et al. Prevalence, incidence, and natural history of simple ovarian cysts among women > 55 years old in a large cancer screening trial. Am J Obstet Gynecol 2010; 202(4):373.e1–e9. doi:10.1016/j.ajog.2009.11.029
  7. American College of Obstetricians and Gynecologists’ Committee on Practice Bulletins—Gynecology. Practice Bulletin No. 174: Evaluation and Management of Adnexal Masses. Obstet Gynecol 2016; 128(5):e210-226. doi:10.1097/AOG.0000000000001768
  8. Royal College of Obstetricians and Gynaecologists. The management of ovarian cysts in postmenopausal women. Green-top guideline 2016; 34:1–31. www.rcog.org.uk/globalassets/documents/guidelines/green-top-guidelines/gtg_34.pdf. Accessed August 16, 2018.
  9. American College of Radiology Choosing Wisely. Imaging tests for ovarian cysts. When you need an ultrasound—and when you don’t. www.choosingwisely.org/wp-content/uploads/2012/09/ChoosingWiselyOvarianCystsACR-ER_Update.pdf. Accessed August 16, 2018.
  10. Levine D, Brown DL, Andreotti RF, et al. Management of asymptomatic ovarian and other adnexal cysts imaged at US: Society of Radiologists in Ultrasound Consensus Conference Statement. Radiology 2010; 256(3):943–954. doi:10.1148/radiol.10100213
References
  1. van Nagell JR Jr, Miiler RW. Evaluation and management of ultrasonographically detected ovarian tumors in asymptomatic women. Obstet Gynecol 2016; 127(5):848–858. doi:10.1097/AOG.0000000000001384
  2. Pavlik EJ, Ueland FR, Miller RW, et al. Frequency and disposition of ovarian abnormalities followed with serial transvaginal ultrasonography. Obstet Gynecol 2013; 122(2 pt 1):210–217. doi:10.1097/AOG.0b013e318298def5
  3. Royal College of Obstetricians and Gynaecologists. Management of suspected ovarian masses in premenopausal women. Green-top guideline 2011; 62:1–14. www.rcog.org.uk/globalassets/documents/guidelines/gtg_62.pdf. Accessed August 16, 2018.
  4. Timmerman D, Testa AC, Bourne T, et al. Simple ultrasound-based rules for the diagnosis of ovarian cancer. Ultrasound Obstet Gynecol 2008; 31(6):681–690. doi:10.1002/uog.5365
  5. Glanc P, Benacerraf B, Bourne T, et al. First international consensus report on adnexal masses: management recommendations. J Ultrasound Med 2017; 36(5):849–863. doi:10.1002/jum.14197
  6. Greenlee RT, Kessel B, Williams CR, et al. Prevalence, incidence, and natural history of simple ovarian cysts among women > 55 years old in a large cancer screening trial. Am J Obstet Gynecol 2010; 202(4):373.e1–e9. doi:10.1016/j.ajog.2009.11.029
  7. American College of Obstetricians and Gynecologists’ Committee on Practice Bulletins—Gynecology. Practice Bulletin No. 174: Evaluation and Management of Adnexal Masses. Obstet Gynecol 2016; 128(5):e210-226. doi:10.1097/AOG.0000000000001768
  8. Royal College of Obstetricians and Gynaecologists. The management of ovarian cysts in postmenopausal women. Green-top guideline 2016; 34:1–31. www.rcog.org.uk/globalassets/documents/guidelines/green-top-guidelines/gtg_34.pdf. Accessed August 16, 2018.
  9. American College of Radiology Choosing Wisely. Imaging tests for ovarian cysts. When you need an ultrasound—and when you don’t. www.choosingwisely.org/wp-content/uploads/2012/09/ChoosingWiselyOvarianCystsACR-ER_Update.pdf. Accessed August 16, 2018.
  10. Levine D, Brown DL, Andreotti RF, et al. Management of asymptomatic ovarian and other adnexal cysts imaged at US: Society of Radiologists in Ultrasound Consensus Conference Statement. Radiology 2010; 256(3):943–954. doi:10.1148/radiol.10100213
Issue
Cleveland Clinic Journal of Medicine - 85(10)
Issue
Cleveland Clinic Journal of Medicine - 85(10)
Page Number
745-747
Page Number
745-747
Publications
Publications
Topics
Article Type
Display Headline
How long should we follow simple ovarian cysts with pelvic ultrasonography?
Display Headline
How long should we follow simple ovarian cysts with pelvic ultrasonography?
Legacy Keywords
ovarian cysts, ultrasonography, ultrasound, cancer antigen 125, CA-125, Megan McNamara, Rachel Brook
Legacy Keywords
ovarian cysts, ultrasonography, ultrasound, cancer antigen 125, CA-125, Megan McNamara, Rachel Brook
Sections
Disallow All Ads
Content Gating
No Gating (article Unlocked/Free)
Alternative CME
Disqus Comments
Default
Gate On Date
Wed, 09/26/2018 - 11:45
Un-Gate On Date
Wed, 09/26/2018 - 11:45
Use ProPublica
CFC Schedule Remove Status
Wed, 09/26/2018 - 11:45
Article PDF Media

Is Pap testing still needed after hysterectomy?

Article Type
Changed
Mon, 06/25/2018 - 14:07
Display Headline
Is Pap testing still needed after hysterectomy?

A 50-year-old woman presents for a new patient visit. She underwent vaginal hysterectomy for menorrhagia 4 years ago, with removal of the uterus and cervix. Tissue studies at that time were negative for dysplasia. Her previous physician performed routine Papanicolaou (Pap) tests, and she asks you to continue this screening. How do you counsel her about Pap testing after hysterectomy for benign disease?

SCREENING GUIDELINES

Introduced in 1941, the Pap test is an example of a successful screening tool, improving detection of early cervical cancer and reducing rates of morbidity and death due to cervical cancer. Early stages of cervical cancer are the most curable.1

Screening in women who have a cervix

In 2012, the US Preventive Services Task Force (USPSTF) updated its 2003 recommendations for cervical cancer screening.1 In the same year, the American Cancer Society, the American Society for Colposcopy and Cervical Pathology, and the American Society for Clinical Pathology published a consensus guideline.2 This was followed by publication of a guideline from the American College of Obstetricians and Gynecologists.3 These guidelines all recommend Pap testing for cervical cancer every 3 years in women ages 21 to 65. In women ages 30 to 65, the screening interval can be lengthened to every 5 years if the patient undergoes cotesting for human papillomavirus (HPV). These recommendations apply only to women with a cervix.

No screening after hysterectomy for benign indications

Women who undergo hysterectomy with complete removal of the cervix for benign indications, ie, for reasons other than malignancy, are no longer at risk of cervical cancer. Pap testing could still detect vaginal cancer, but vaginal cancer is rare and screening for it is not indicated. The USPSTF 2003 and 2012 guidelines recommend not performing Pap testing in women who had had a hysterectomy for benign indications.1

Vaginal cancer is rare

Although cervical and vaginal cancers share risk factors, vaginal cancer accounts for only 0.3% of all invasive cancers and 1% to 2% of all gynecologic malignancies in the United States.4

A review of 39 population-based cancer registries from 1998 to 2003 found the incidence rate for in situ vaginal cancer to be 0.18 per 100,000 women, and the incidence rate for invasive vaginal cancer was 0.69 per 100,000. Rates were higher in older women and in certain ethnic and racial groups, including black and Hispanic women.4

When the cervix is removed during hysterectomy for a benign indication, the patient’s risk of vaginal cancer or its precursors is extremely low. Pearce et al5 reviewed Pap tests obtained from the vaginal cuff in 6,265 women who had undergone hysterectomy for benign disease. Their 2-year study reviewed 9,610 vaginal Pap tests, and in only 5 women was vaginal intraepithelial neoplasia type I or II found, and none of the 5 had biopsy-proven vaginal cancer. Only 1.1% of all Pap tests were abnormal. The authors concluded that the positive predictive value for detecting vaginal cancer was 0%.5

A retrospective study by Piscitelli et al6 in 1995 looked back 10 years and found an extremely low incidence of vaginal dysplasia in women who had undergone hysterectomy for a benign indication. Their findings, coupled with the high rate of false-positive tests, do not support cytologic screening of the vagina after hysterectomy for a benign indication. The data also suggested that 633 tests would need to be performed to diagnose 1 case of vaginal dysplasia.6 Other studies have also reported a low yield of vaginal cuff cytologic testing after hysterectomy for benign disease.

Therefore, given the low prevalence of disease and the lack of evidence of benefit of screening after hysterectomy for benign indications, Pap testing of the vaginal cuff is not recommended in these patients.7

 

 

Screening for women at high risk after hysterectomy

For women with a history of grade 2 or 3 cervical intraepithelial neoplasia who have undergone hysterectomy, there are only limited data on subsequent disease risk.

Wiener et al8 followed 193 post-hysterectomy patients who had a history of cervical intraepithelial neoplasia with Pap testing annually for more than 10 years for a total of 2,800 years of follow-up. The estimated incidence of abnormal cytology (0.7/1,000) was higher than in the general population.8

Thus, for these women and for others at high risk who have undergone hysterectomy and have a previous diagnosis of cervical cancer, who had been exposed to diethylstilbestrol, or who are immunocompromised, Pap testing to screen for cancer in the vaginal cuff is recommended, as they are at higher risk of dysplasia at the vaginal cuff.2

PRACTICE TRENDS, AREAS FOR IMPROVEMENT

Despite recommendations against screening, many providers continue this non-evidence-based practice.4

The 2000–2013 National Health Interview Survey of women age 20 or older who had undergone hysterectomy asked about their most recent Pap test by self-report. Women were excluded if they had a history of cervical cancer, if they had had a Pap test for another health problem, or if the result of the recent Pap test was not known. In 2000, nearly half (49.1%) of the respondents said they had received a Pap test in the previous year; in 2013, the percentage undergoing testing was down to 32.1%, but testing was unnecessary in 22.1%. Screening was largely due to clinician recommendations, but it was initiated by patients without clinician recommendations in about one-fourth of cases.9 Lack of knowledge of the revised 2012 guidelines was cited as the primary reason for unnecessary screening.10

A study of provider attitudes toward the cancer screening guidelines cited several reasons for nonadherence: patient concern about the guidelines; quality metrics that are incongruent with the guidelines; provider disagreement with the guidelines; risk of malpractice litigation; and lack of time to discuss the guidelines with patients.11

As the healthcare landscape changes to team-based care, the clinician and the entire healthcare team should educate patients about the role of vaginal cancer screening after hysterectomy for benign reasons. Given the limited time clinicians have with patients during an office visit, innovative tools and systems outside the office are needed to educate patients about the risks and benefits of screening.11 And notices in the electronic medical record may help busy clinicians keep up with current guidelines.10

THE CLINICAL BOTTOM LINE

Pap testing to screen for vaginal cancer in women who have undergone hysterectomy for a benign indication is an example of more testing, not better care. Evidence is lacking to justify this test in women who are not at high risk of cervical cancer. To reduce the overuse of cytology screening tests, providers need to stay informed about evidence-based best practices and and to pass this information along to patients.

We should focus our resources on HPV vaccination and outreach to increase screening efforts in geographic areas with low rates of Pap testing rather than provide unnecessary Pap testing for women who have undergone hysterectomy for a benign indication.

References
  1. Moyer VA; US Preventive Services Task Force. Screening for cervical cancer: US Preventive Services Task Force recommendation statement. Ann Intern Med 2012; 156(11):880–891, W312. doi:10.7326/0003-4819-156-12-201206190-00424
  2. Saslow D, Solomon D, Lawson HW, et al; American Cancer Society; American Society for Colposcopy and Cervical Pathology; American Society for Clinical Pathology. American Cancer Society, American Society for Colposcopy and Cervical Pathology, and American Society for Clinical Pathology screening guidelines for the prevention and early detection of cervical cancer. Am J Clin Pathol 2012; 137(4):516–542. doi:10.1309/AJCPTGD94EVRSJCG
  3. Committee on Practice Bulletins—Gynecology. ACOG practice bulletin number 131: screening for cervical cancer. Obstet Gynecol 2012; 120(5):1222–1238. doi:10.1097/AOG.0b013e318277c92a
  4. Wu X, Matanoski G, Chen VW, et al. Descriptive epidemiology of vaginal cancer incidence and survival by race, ethnicity, and age in the United States. Cancer 2008; 113(10 suppl):2873–2882. doi:10.1002/cncr.23757
  5. Pearce KF, Haefner HK, Sarwar SF, Nolan TE. Cytopathological findings on vaginal Papanicolaou smears after hysterectomy for benign gynecologic disease. N Engl J Med 1996; 335(21):1559–1562. doi:10.1056/NEJM199611213352103
  6. Piscitelli JT, Bastian LA, Wilkes A, Simel DL. Cytologic screening after hysterectomy for benign disease. Am J Obstet Gynecol 1995;173(2):424–432. pmid:7645617
  7. Stokes-Lampard H, Wilson S, Waddell C, Ryan A, Holder R, Kehoe S. Vaginal vault smears after hysterectomy for reasons other than malignancy: a systematic review of the literature. BJOG 2006; 113(12):1354–1365. doi:10.1111/j.1471-0528.2006.01099.x
  8. Wiener JJ, Sweetnam PM, Jones JM. Long term follow up of women after hysterectomy with a history of pre-invasive cancer of the cervix. Br J Obstet Gynaecol 1992; 99(11):907–910. pmid:1450141
  9. Guo F, Kuo YF. Roles of health care providers and patients in initiation of unnecessary Papanicolaou testing after total hysterectomy. Am J Public Health 2016; 106(11):2005–2011. doi:10.2105/AJPH.2016.303360
  10. Teoh DG, Marriott AE, Isaksson Vogel R, et al. Adherence to the 2012 national cervical cancer screening guidelines: a pilot study. Am J Obstet Gynecol 2015; 212(1):62.e1–e9. doi:10.1016/j.ajog.2014.06.057
  11. Haas JS, Sprague BL, Klabunde CN, et al; PROSPR (Population-based Research Optimizing Screening through Personalized Regimens) Consortium. Provider attitudes and screening practices following changes in breast and cervical cancer screening guidelines. J Gen Intern Med 2016; 31(1):52–59.  doi:10.1007/s11606-015-3449-5
Article PDF
Author and Disclosure Information

Halle G. Sobel, MD
Associate Professor of Medicine, General Internal Medicine and Geriatrics, University of Vermont Medical Center, Burlington

Elise Everett, MD
Associate Professor, Department of Obstetrics and Gynecology, University of Vermont Medical Center, Burlington

Laura D. Lipold, MD
Director, Primary Care Women’s Health, Department of Family Medicine, Cleveland Clinic

Address: Halle G. Sobel, MD, General Internal Medicine and Geriatrics, University of Vermont Medical Center, 1 South Prospect Street, Burlington, VT 05401; halle.sobel@uvmhealth.org; halle.sobel@gmail.com

Issue
Cleveland Clinic Journal of Medicine - 85(6)
Publications
Topics
Page Number
456-458
Legacy Keywords
Papanicolaou, Pap test, cervical cancer, cytology, hysterectomy, screening, testing, Halle Sobel, Elise Everett, Laura Lipold
Sections
Author and Disclosure Information

Halle G. Sobel, MD
Associate Professor of Medicine, General Internal Medicine and Geriatrics, University of Vermont Medical Center, Burlington

Elise Everett, MD
Associate Professor, Department of Obstetrics and Gynecology, University of Vermont Medical Center, Burlington

Laura D. Lipold, MD
Director, Primary Care Women’s Health, Department of Family Medicine, Cleveland Clinic

Address: Halle G. Sobel, MD, General Internal Medicine and Geriatrics, University of Vermont Medical Center, 1 South Prospect Street, Burlington, VT 05401; halle.sobel@uvmhealth.org; halle.sobel@gmail.com

Author and Disclosure Information

Halle G. Sobel, MD
Associate Professor of Medicine, General Internal Medicine and Geriatrics, University of Vermont Medical Center, Burlington

Elise Everett, MD
Associate Professor, Department of Obstetrics and Gynecology, University of Vermont Medical Center, Burlington

Laura D. Lipold, MD
Director, Primary Care Women’s Health, Department of Family Medicine, Cleveland Clinic

Address: Halle G. Sobel, MD, General Internal Medicine and Geriatrics, University of Vermont Medical Center, 1 South Prospect Street, Burlington, VT 05401; halle.sobel@uvmhealth.org; halle.sobel@gmail.com

Article PDF
Article PDF
Related Articles

A 50-year-old woman presents for a new patient visit. She underwent vaginal hysterectomy for menorrhagia 4 years ago, with removal of the uterus and cervix. Tissue studies at that time were negative for dysplasia. Her previous physician performed routine Papanicolaou (Pap) tests, and she asks you to continue this screening. How do you counsel her about Pap testing after hysterectomy for benign disease?

SCREENING GUIDELINES

Introduced in 1941, the Pap test is an example of a successful screening tool, improving detection of early cervical cancer and reducing rates of morbidity and death due to cervical cancer. Early stages of cervical cancer are the most curable.1

Screening in women who have a cervix

In 2012, the US Preventive Services Task Force (USPSTF) updated its 2003 recommendations for cervical cancer screening.1 In the same year, the American Cancer Society, the American Society for Colposcopy and Cervical Pathology, and the American Society for Clinical Pathology published a consensus guideline.2 This was followed by publication of a guideline from the American College of Obstetricians and Gynecologists.3 These guidelines all recommend Pap testing for cervical cancer every 3 years in women ages 21 to 65. In women ages 30 to 65, the screening interval can be lengthened to every 5 years if the patient undergoes cotesting for human papillomavirus (HPV). These recommendations apply only to women with a cervix.

No screening after hysterectomy for benign indications

Women who undergo hysterectomy with complete removal of the cervix for benign indications, ie, for reasons other than malignancy, are no longer at risk of cervical cancer. Pap testing could still detect vaginal cancer, but vaginal cancer is rare and screening for it is not indicated. The USPSTF 2003 and 2012 guidelines recommend not performing Pap testing in women who had had a hysterectomy for benign indications.1

Vaginal cancer is rare

Although cervical and vaginal cancers share risk factors, vaginal cancer accounts for only 0.3% of all invasive cancers and 1% to 2% of all gynecologic malignancies in the United States.4

A review of 39 population-based cancer registries from 1998 to 2003 found the incidence rate for in situ vaginal cancer to be 0.18 per 100,000 women, and the incidence rate for invasive vaginal cancer was 0.69 per 100,000. Rates were higher in older women and in certain ethnic and racial groups, including black and Hispanic women.4

When the cervix is removed during hysterectomy for a benign indication, the patient’s risk of vaginal cancer or its precursors is extremely low. Pearce et al5 reviewed Pap tests obtained from the vaginal cuff in 6,265 women who had undergone hysterectomy for benign disease. Their 2-year study reviewed 9,610 vaginal Pap tests, and in only 5 women was vaginal intraepithelial neoplasia type I or II found, and none of the 5 had biopsy-proven vaginal cancer. Only 1.1% of all Pap tests were abnormal. The authors concluded that the positive predictive value for detecting vaginal cancer was 0%.5

A retrospective study by Piscitelli et al6 in 1995 looked back 10 years and found an extremely low incidence of vaginal dysplasia in women who had undergone hysterectomy for a benign indication. Their findings, coupled with the high rate of false-positive tests, do not support cytologic screening of the vagina after hysterectomy for a benign indication. The data also suggested that 633 tests would need to be performed to diagnose 1 case of vaginal dysplasia.6 Other studies have also reported a low yield of vaginal cuff cytologic testing after hysterectomy for benign disease.

Therefore, given the low prevalence of disease and the lack of evidence of benefit of screening after hysterectomy for benign indications, Pap testing of the vaginal cuff is not recommended in these patients.7

 

 

Screening for women at high risk after hysterectomy

For women with a history of grade 2 or 3 cervical intraepithelial neoplasia who have undergone hysterectomy, there are only limited data on subsequent disease risk.

Wiener et al8 followed 193 post-hysterectomy patients who had a history of cervical intraepithelial neoplasia with Pap testing annually for more than 10 years for a total of 2,800 years of follow-up. The estimated incidence of abnormal cytology (0.7/1,000) was higher than in the general population.8

Thus, for these women and for others at high risk who have undergone hysterectomy and have a previous diagnosis of cervical cancer, who had been exposed to diethylstilbestrol, or who are immunocompromised, Pap testing to screen for cancer in the vaginal cuff is recommended, as they are at higher risk of dysplasia at the vaginal cuff.2

PRACTICE TRENDS, AREAS FOR IMPROVEMENT

Despite recommendations against screening, many providers continue this non-evidence-based practice.4

The 2000–2013 National Health Interview Survey of women age 20 or older who had undergone hysterectomy asked about their most recent Pap test by self-report. Women were excluded if they had a history of cervical cancer, if they had had a Pap test for another health problem, or if the result of the recent Pap test was not known. In 2000, nearly half (49.1%) of the respondents said they had received a Pap test in the previous year; in 2013, the percentage undergoing testing was down to 32.1%, but testing was unnecessary in 22.1%. Screening was largely due to clinician recommendations, but it was initiated by patients without clinician recommendations in about one-fourth of cases.9 Lack of knowledge of the revised 2012 guidelines was cited as the primary reason for unnecessary screening.10

A study of provider attitudes toward the cancer screening guidelines cited several reasons for nonadherence: patient concern about the guidelines; quality metrics that are incongruent with the guidelines; provider disagreement with the guidelines; risk of malpractice litigation; and lack of time to discuss the guidelines with patients.11

As the healthcare landscape changes to team-based care, the clinician and the entire healthcare team should educate patients about the role of vaginal cancer screening after hysterectomy for benign reasons. Given the limited time clinicians have with patients during an office visit, innovative tools and systems outside the office are needed to educate patients about the risks and benefits of screening.11 And notices in the electronic medical record may help busy clinicians keep up with current guidelines.10

THE CLINICAL BOTTOM LINE

Pap testing to screen for vaginal cancer in women who have undergone hysterectomy for a benign indication is an example of more testing, not better care. Evidence is lacking to justify this test in women who are not at high risk of cervical cancer. To reduce the overuse of cytology screening tests, providers need to stay informed about evidence-based best practices and and to pass this information along to patients.

We should focus our resources on HPV vaccination and outreach to increase screening efforts in geographic areas with low rates of Pap testing rather than provide unnecessary Pap testing for women who have undergone hysterectomy for a benign indication.

A 50-year-old woman presents for a new patient visit. She underwent vaginal hysterectomy for menorrhagia 4 years ago, with removal of the uterus and cervix. Tissue studies at that time were negative for dysplasia. Her previous physician performed routine Papanicolaou (Pap) tests, and she asks you to continue this screening. How do you counsel her about Pap testing after hysterectomy for benign disease?

SCREENING GUIDELINES

Introduced in 1941, the Pap test is an example of a successful screening tool, improving detection of early cervical cancer and reducing rates of morbidity and death due to cervical cancer. Early stages of cervical cancer are the most curable.1

Screening in women who have a cervix

In 2012, the US Preventive Services Task Force (USPSTF) updated its 2003 recommendations for cervical cancer screening.1 In the same year, the American Cancer Society, the American Society for Colposcopy and Cervical Pathology, and the American Society for Clinical Pathology published a consensus guideline.2 This was followed by publication of a guideline from the American College of Obstetricians and Gynecologists.3 These guidelines all recommend Pap testing for cervical cancer every 3 years in women ages 21 to 65. In women ages 30 to 65, the screening interval can be lengthened to every 5 years if the patient undergoes cotesting for human papillomavirus (HPV). These recommendations apply only to women with a cervix.

No screening after hysterectomy for benign indications

Women who undergo hysterectomy with complete removal of the cervix for benign indications, ie, for reasons other than malignancy, are no longer at risk of cervical cancer. Pap testing could still detect vaginal cancer, but vaginal cancer is rare and screening for it is not indicated. The USPSTF 2003 and 2012 guidelines recommend not performing Pap testing in women who had had a hysterectomy for benign indications.1

Vaginal cancer is rare

Although cervical and vaginal cancers share risk factors, vaginal cancer accounts for only 0.3% of all invasive cancers and 1% to 2% of all gynecologic malignancies in the United States.4

A review of 39 population-based cancer registries from 1998 to 2003 found the incidence rate for in situ vaginal cancer to be 0.18 per 100,000 women, and the incidence rate for invasive vaginal cancer was 0.69 per 100,000. Rates were higher in older women and in certain ethnic and racial groups, including black and Hispanic women.4

When the cervix is removed during hysterectomy for a benign indication, the patient’s risk of vaginal cancer or its precursors is extremely low. Pearce et al5 reviewed Pap tests obtained from the vaginal cuff in 6,265 women who had undergone hysterectomy for benign disease. Their 2-year study reviewed 9,610 vaginal Pap tests, and in only 5 women was vaginal intraepithelial neoplasia type I or II found, and none of the 5 had biopsy-proven vaginal cancer. Only 1.1% of all Pap tests were abnormal. The authors concluded that the positive predictive value for detecting vaginal cancer was 0%.5

A retrospective study by Piscitelli et al6 in 1995 looked back 10 years and found an extremely low incidence of vaginal dysplasia in women who had undergone hysterectomy for a benign indication. Their findings, coupled with the high rate of false-positive tests, do not support cytologic screening of the vagina after hysterectomy for a benign indication. The data also suggested that 633 tests would need to be performed to diagnose 1 case of vaginal dysplasia.6 Other studies have also reported a low yield of vaginal cuff cytologic testing after hysterectomy for benign disease.

Therefore, given the low prevalence of disease and the lack of evidence of benefit of screening after hysterectomy for benign indications, Pap testing of the vaginal cuff is not recommended in these patients.7

 

 

Screening for women at high risk after hysterectomy

For women with a history of grade 2 or 3 cervical intraepithelial neoplasia who have undergone hysterectomy, there are only limited data on subsequent disease risk.

Wiener et al8 followed 193 post-hysterectomy patients who had a history of cervical intraepithelial neoplasia with Pap testing annually for more than 10 years for a total of 2,800 years of follow-up. The estimated incidence of abnormal cytology (0.7/1,000) was higher than in the general population.8

Thus, for these women and for others at high risk who have undergone hysterectomy and have a previous diagnosis of cervical cancer, who had been exposed to diethylstilbestrol, or who are immunocompromised, Pap testing to screen for cancer in the vaginal cuff is recommended, as they are at higher risk of dysplasia at the vaginal cuff.2

PRACTICE TRENDS, AREAS FOR IMPROVEMENT

Despite recommendations against screening, many providers continue this non-evidence-based practice.4

The 2000–2013 National Health Interview Survey of women age 20 or older who had undergone hysterectomy asked about their most recent Pap test by self-report. Women were excluded if they had a history of cervical cancer, if they had had a Pap test for another health problem, or if the result of the recent Pap test was not known. In 2000, nearly half (49.1%) of the respondents said they had received a Pap test in the previous year; in 2013, the percentage undergoing testing was down to 32.1%, but testing was unnecessary in 22.1%. Screening was largely due to clinician recommendations, but it was initiated by patients without clinician recommendations in about one-fourth of cases.9 Lack of knowledge of the revised 2012 guidelines was cited as the primary reason for unnecessary screening.10

A study of provider attitudes toward the cancer screening guidelines cited several reasons for nonadherence: patient concern about the guidelines; quality metrics that are incongruent with the guidelines; provider disagreement with the guidelines; risk of malpractice litigation; and lack of time to discuss the guidelines with patients.11

As the healthcare landscape changes to team-based care, the clinician and the entire healthcare team should educate patients about the role of vaginal cancer screening after hysterectomy for benign reasons. Given the limited time clinicians have with patients during an office visit, innovative tools and systems outside the office are needed to educate patients about the risks and benefits of screening.11 And notices in the electronic medical record may help busy clinicians keep up with current guidelines.10

THE CLINICAL BOTTOM LINE

Pap testing to screen for vaginal cancer in women who have undergone hysterectomy for a benign indication is an example of more testing, not better care. Evidence is lacking to justify this test in women who are not at high risk of cervical cancer. To reduce the overuse of cytology screening tests, providers need to stay informed about evidence-based best practices and and to pass this information along to patients.

We should focus our resources on HPV vaccination and outreach to increase screening efforts in geographic areas with low rates of Pap testing rather than provide unnecessary Pap testing for women who have undergone hysterectomy for a benign indication.

References
  1. Moyer VA; US Preventive Services Task Force. Screening for cervical cancer: US Preventive Services Task Force recommendation statement. Ann Intern Med 2012; 156(11):880–891, W312. doi:10.7326/0003-4819-156-12-201206190-00424
  2. Saslow D, Solomon D, Lawson HW, et al; American Cancer Society; American Society for Colposcopy and Cervical Pathology; American Society for Clinical Pathology. American Cancer Society, American Society for Colposcopy and Cervical Pathology, and American Society for Clinical Pathology screening guidelines for the prevention and early detection of cervical cancer. Am J Clin Pathol 2012; 137(4):516–542. doi:10.1309/AJCPTGD94EVRSJCG
  3. Committee on Practice Bulletins—Gynecology. ACOG practice bulletin number 131: screening for cervical cancer. Obstet Gynecol 2012; 120(5):1222–1238. doi:10.1097/AOG.0b013e318277c92a
  4. Wu X, Matanoski G, Chen VW, et al. Descriptive epidemiology of vaginal cancer incidence and survival by race, ethnicity, and age in the United States. Cancer 2008; 113(10 suppl):2873–2882. doi:10.1002/cncr.23757
  5. Pearce KF, Haefner HK, Sarwar SF, Nolan TE. Cytopathological findings on vaginal Papanicolaou smears after hysterectomy for benign gynecologic disease. N Engl J Med 1996; 335(21):1559–1562. doi:10.1056/NEJM199611213352103
  6. Piscitelli JT, Bastian LA, Wilkes A, Simel DL. Cytologic screening after hysterectomy for benign disease. Am J Obstet Gynecol 1995;173(2):424–432. pmid:7645617
  7. Stokes-Lampard H, Wilson S, Waddell C, Ryan A, Holder R, Kehoe S. Vaginal vault smears after hysterectomy for reasons other than malignancy: a systematic review of the literature. BJOG 2006; 113(12):1354–1365. doi:10.1111/j.1471-0528.2006.01099.x
  8. Wiener JJ, Sweetnam PM, Jones JM. Long term follow up of women after hysterectomy with a history of pre-invasive cancer of the cervix. Br J Obstet Gynaecol 1992; 99(11):907–910. pmid:1450141
  9. Guo F, Kuo YF. Roles of health care providers and patients in initiation of unnecessary Papanicolaou testing after total hysterectomy. Am J Public Health 2016; 106(11):2005–2011. doi:10.2105/AJPH.2016.303360
  10. Teoh DG, Marriott AE, Isaksson Vogel R, et al. Adherence to the 2012 national cervical cancer screening guidelines: a pilot study. Am J Obstet Gynecol 2015; 212(1):62.e1–e9. doi:10.1016/j.ajog.2014.06.057
  11. Haas JS, Sprague BL, Klabunde CN, et al; PROSPR (Population-based Research Optimizing Screening through Personalized Regimens) Consortium. Provider attitudes and screening practices following changes in breast and cervical cancer screening guidelines. J Gen Intern Med 2016; 31(1):52–59.  doi:10.1007/s11606-015-3449-5
References
  1. Moyer VA; US Preventive Services Task Force. Screening for cervical cancer: US Preventive Services Task Force recommendation statement. Ann Intern Med 2012; 156(11):880–891, W312. doi:10.7326/0003-4819-156-12-201206190-00424
  2. Saslow D, Solomon D, Lawson HW, et al; American Cancer Society; American Society for Colposcopy and Cervical Pathology; American Society for Clinical Pathology. American Cancer Society, American Society for Colposcopy and Cervical Pathology, and American Society for Clinical Pathology screening guidelines for the prevention and early detection of cervical cancer. Am J Clin Pathol 2012; 137(4):516–542. doi:10.1309/AJCPTGD94EVRSJCG
  3. Committee on Practice Bulletins—Gynecology. ACOG practice bulletin number 131: screening for cervical cancer. Obstet Gynecol 2012; 120(5):1222–1238. doi:10.1097/AOG.0b013e318277c92a
  4. Wu X, Matanoski G, Chen VW, et al. Descriptive epidemiology of vaginal cancer incidence and survival by race, ethnicity, and age in the United States. Cancer 2008; 113(10 suppl):2873–2882. doi:10.1002/cncr.23757
  5. Pearce KF, Haefner HK, Sarwar SF, Nolan TE. Cytopathological findings on vaginal Papanicolaou smears after hysterectomy for benign gynecologic disease. N Engl J Med 1996; 335(21):1559–1562. doi:10.1056/NEJM199611213352103
  6. Piscitelli JT, Bastian LA, Wilkes A, Simel DL. Cytologic screening after hysterectomy for benign disease. Am J Obstet Gynecol 1995;173(2):424–432. pmid:7645617
  7. Stokes-Lampard H, Wilson S, Waddell C, Ryan A, Holder R, Kehoe S. Vaginal vault smears after hysterectomy for reasons other than malignancy: a systematic review of the literature. BJOG 2006; 113(12):1354–1365. doi:10.1111/j.1471-0528.2006.01099.x
  8. Wiener JJ, Sweetnam PM, Jones JM. Long term follow up of women after hysterectomy with a history of pre-invasive cancer of the cervix. Br J Obstet Gynaecol 1992; 99(11):907–910. pmid:1450141
  9. Guo F, Kuo YF. Roles of health care providers and patients in initiation of unnecessary Papanicolaou testing after total hysterectomy. Am J Public Health 2016; 106(11):2005–2011. doi:10.2105/AJPH.2016.303360
  10. Teoh DG, Marriott AE, Isaksson Vogel R, et al. Adherence to the 2012 national cervical cancer screening guidelines: a pilot study. Am J Obstet Gynecol 2015; 212(1):62.e1–e9. doi:10.1016/j.ajog.2014.06.057
  11. Haas JS, Sprague BL, Klabunde CN, et al; PROSPR (Population-based Research Optimizing Screening through Personalized Regimens) Consortium. Provider attitudes and screening practices following changes in breast and cervical cancer screening guidelines. J Gen Intern Med 2016; 31(1):52–59.  doi:10.1007/s11606-015-3449-5
Issue
Cleveland Clinic Journal of Medicine - 85(6)
Issue
Cleveland Clinic Journal of Medicine - 85(6)
Page Number
456-458
Page Number
456-458
Publications
Publications
Topics
Article Type
Display Headline
Is Pap testing still needed after hysterectomy?
Display Headline
Is Pap testing still needed after hysterectomy?
Legacy Keywords
Papanicolaou, Pap test, cervical cancer, cytology, hysterectomy, screening, testing, Halle Sobel, Elise Everett, Laura Lipold
Legacy Keywords
Papanicolaou, Pap test, cervical cancer, cytology, hysterectomy, screening, testing, Halle Sobel, Elise Everett, Laura Lipold
Sections
Disallow All Ads
Content Gating
No Gating (article Unlocked/Free)
Alternative CME
Disqus Comments
Default
Gate On Date
Tue, 05/29/2018 - 10:15
Un-Gate On Date
Tue, 05/29/2018 - 10:15
Use ProPublica
CFC Schedule Remove Status
Tue, 05/29/2018 - 10:15
Article PDF Media

Measuring both serum amylase and lipase for acute pancreatitis lowers quality and raises cost

Article Type
Changed
Fri, 09/01/2017 - 08:48
Display Headline
Measuring both serum amylase and lipase for acute pancreatitis lowers quality and raises cost

A 43-year-old, previously healthy woman was admitted to the hospital after 1 day of severe epigastric abdominal pain, nausea, and vomiting. She denied alcohol or tobacco use.

Her physical examination revealed normal vital signs and epigastric tenderness without rebound tenderness.

Notable laboratory results:

  • Aspartate aminotransferase 149 U/L (reference range 10–35)
  • Alanine aminotransferase 140 U/L (10–35)
  • Alkaline phosphatase 178 IU/L (35–104)
  • Total bilirubin 1.8 mg/dL (0.2–1.3)
  • Amylase 1,244 U/L (28–100)
  • Lipase 14,628 U/L (7–59).

Abdominal ultrasonography showed a dilated bile duct and gallstones.

The patient was diagnosed with biliary pancreatitis and was treated by placing her on nothing-by-mouth (NPO) status and giving intravenous fluids and analgesics. All symptoms had resolved by hospital day 3. She underwent laparoscopic cholecystectomy and was discharged the following day.

This is a typical case of biliary pancreatitis that was diagnosed and treated appropriately with a positive outcome. But was it necessary or beneficial to measure both the serum amylase and serum lipase to make the correct diagnosis and treat the patient appropriately?

IS MEASURING SERUM AMYLASE NECESSARY?

The American College of Gastroenterology practice guidelines suggest that measuring both serum amylase and serum lipase is not necessary.1 Serum lipase alone is the preferred test for diagnosing acute pancreatitis, since it is more sensitive than serum amylase, just as specific, rises more quickly, and remains elevated longer.

In a retrospective study of 151 patients with acute pancreatitis,2 the sensitivity of lipase was 96.6% and the specificity was 99.4%.2 In contrast, the sensitivity of amylase was 78.6% and the specificity was 99.1%.

In another study,3 in 476 patients with acute pancreatitis, lipase had a sensitivity of 91% vs 62% for amylase. Again, specificity was similar between the two tests (92% for lipase and 93% for amylase). The authors concluded that lipase should replace amylase as the first-line laboratory investigation for suspected acute pancreatitis.

Smith et al4 reviewed 1,825 patients with acute pancreatitis and similarly concluded that pancreatic lipase is a more accurate biomarker of acute pancreatitis than serum amylase.

PRACTICE AT OUR HOSPITAL

Despite this guideline and evidence, concurrent ordering of serum amylase and lipase is common at many institutions.

We evaluated the practice of ordering both serum amylase and lipase for diagnosis of acute pancreatitis at our 300-bed academic medical hospital. From January 2011 through August 2014, our institution completed 26,254 orders for serum amylase and lipase measurement in 13,198 patients. In 9,938 (75%) of the patients, amylase and lipase were ordered concurrently. Of these, 482 patients (4.8%) had either amylase or lipase elevated above the diagnostic threshold, ie, 3 times the upper limit of normal, and 63 of the 482 patients had an elevation in serum amylase greater than 3 times the upper limit of normal without an elevation in serum lipase.

None of the patients had acute pancreatitis clinically (eg, typical abdominal pain, nausea, vomiting) or on imaging (pancreatic edema). The definitive cause of nonpancreatic hyperamylasemia could not be determined in these patients; they did not have evidence of salivary disorder, malignancy, or tubo-ovarian disease, and the hyperamylasemia was believed to be related to renal disease, diabetic ketoacidosis, infection, or medications, or to be idiopathic.

In 12 patients, the discrepancy between an elevated amylase and normal lipase resulted in additional imaging with computed tomography. Four patients were also unnecessarily kept NPO for 1 to 3 days, depriving them of nutrition and prolonging their hospital stay.

To minimize concurrent ordering of serum amylase and lipase, we introduced a best-practice alert in the computerized physician order entry systems. The alert mentioned that “ordering both serum amylase and lipase in cases of suspected pancreatitis is unnecessary. Serum lipase alone is sufficient.” However, ordering providers could still order both tests if they wanted to.

In the 3 months after the alert was implemented, serum lipase was ordered 1,780 times with 532 (30%) concurrent orders of amylase. Before the alert was instituted, amylase testing was ordered a mean of 450 times per month; afterward, this decreased by about 60%.

We are now considering eliminating serum amylase testing, as suggested by prior studies5 and the American Society of Clinical Pathology.6

ELIMINATING NEEDLESS EXPENSES

The relentless and unsustainable rise in healthcare costs has prompted physician-led groups such as the American Board of Internal Medicine Foundation and the American College of Physicians to focus on ways to cut waste and incorporate high-value, cost-conscious care into clinical practice.

In 2009 alone, waste in total healthcare expenditures was estimated at $765 billion. More than half of this astronomical figure was attributed to unnecessary and inefficiently delivered services, expenditures that physicians can directly avoid with changes to their practice.7,8 Unnecessary laboratory tests such as serum amylase are just one of many wasteful practices.

Hospitals have much to lose when unnecessary tests are ordered. For inpatient hospital admissions in the United States, payment is based on the diagnosis-related group system, in which hospitals are paid a fixed amount per diagnosis. There is no additional reimbursement for laboratory tests. An unnecessary test such as serum amylase in suspected cases of acute pancreatitis thus becomes an expense with no corresponding benefit.

The cost of performing a serum amylase test for a typical laboratory is around $4 to $6. Serum amylase testing at our hospital resulted in unnecessary expense of about $35,000 annually. If we add the costs of additional imaging and prolonged hospitalization, the expenses are substantially more.

Despite this, most hospitals have been unwilling or unable to tackle the problem. This may be due to respect for physician autonomy, seemingly small financial loss, or organizational inertia. For the entire healthcare system, these seemingly minor costs add up. For example, from 2011 to 2014, Medicare Part B alone spent $19.4 million on serum amylase testing.

Ordering unnecessary laboratory tests is not a problem specific to our hospital, but rather a common problem encountered at many hospitals. Recognizing the widespread practice of ordering amylase, the Choosing Wisely initiative shared new recommendations from the American Society for Clinical Pathology supporting the use of lipase instead of amylase in suspected acute pancreatitis.7

Physicians who continue to order these tests show a disregard for evidence-based medicine, patient care, and healthcare costs.

CLINICAL BOTTOM LINE

Concurrent use of amylase and lipase testing to diagnose acute pancreatitis is an unnecessary expense for the hospital and can negatively affect patient care as it can lead to additional tests and prolonged hospitalization. Steps should be taken to minimize ordering of amylase by educating physicians and instituting best-practice alerts, or by eliminating the test altogether.

References
  1. Banks PA, Freeman ML, Practice Parameters Committee of the American College of Gastroenterology. Practice guidelines in acute pancreatitis. Am J Gastroenterol 2006; 101:2379–2400.
  2. Gomez D, Addison A, De Rosa A, Brooks A, Cameron IC. Retrospective study of patients with acute pancreatitis: is serum amylase still required? BMJ Open 2012; 2. pii:e001471.
  3. Hofmeyr S, Meyer C, Warren BL. Serum lipase should be the laboratory test of choice for suspected acute pancreatitis. S Afr J Surg 2014; 52:72–75.
  4. Smith RC, Southwell-Keely J, Chesher D. Should serum pancreatic lipase replace serum amylase as a biomarker of acute pancreatitis? ANZ J Surg 2005; 75:399–404.
  5. Volz KA, McGillicuddy DC, Horowitz GL, Wolfe RE, Joyce N, Sanchez LD. Eliminating amylase testing from the evaluation of pancreatitis in the emergency department. West J Emerg Med 2010; 11:344–347.
  6. American Society for Clinical Pathology. Do not test for amylase in cases of suspected pancreatitis. Instead, test for lipase. Choosing Wisely; 2016. www.choosingwisely.org/clinician-lists/american-society-clinical-pathology-testing-for-amylase. Accessed August 3, 2017.
  7. Smith M, Saunders R, Stuckhardt L, McGinnis JM; Committee on the Learning Health Care System in America, eds. Best Care at Lower Cost: The Path to Continuously Learning Health Care in America. Washington, DC: The National Academies Press; 2013. www.hep.fsu.edu/~wahl/artic/NAP/HealthCare13444.pdf. Accessed August 3, 2017.
  8. American College of Physicians. Eliminating healthcare waste and overordering of tests. www.acponline.org/clinical-information/high-value-care/medical-educators-resources/curriculum-for-educators-and-residents/curriculum-version-3. Accessed August 3, 2017.
Article PDF
Author and Disclosure Information

Abdulhadi Akhtar, MD
Department of Surgery, University of Texas Southwestern Medical Center, Dallas, TX

Ravindra Sarode, MD
Professor and Chief, Department of Pathology, University of Texas Southwestern Medical Center, Dallas, TX

Deepak Agrawal, MD
Assistant Professor, Department of Internal Medicine, University of Texas Southwestern Medical Center, Dallas, TX

Address: Deepak Agrawal, MD, Department of Internal Medicine, University of Texas Southwestern Medical Center, 5323 Harry Hines Boulevard, Dallas, TX 75390; Deepak.Agrawal@UTSouthwestern.edu

Issue
Cleveland Clinic Journal of Medicine - 84(9)
Publications
Topics
Page Number
670-672
Legacy Keywords
amylase, lipase, laboratory testing, smart testing, pancreatitis, Abdulhadi Akhtar, Ravindra Sarode, Deepak Agrawal
Sections
Author and Disclosure Information

Abdulhadi Akhtar, MD
Department of Surgery, University of Texas Southwestern Medical Center, Dallas, TX

Ravindra Sarode, MD
Professor and Chief, Department of Pathology, University of Texas Southwestern Medical Center, Dallas, TX

Deepak Agrawal, MD
Assistant Professor, Department of Internal Medicine, University of Texas Southwestern Medical Center, Dallas, TX

Address: Deepak Agrawal, MD, Department of Internal Medicine, University of Texas Southwestern Medical Center, 5323 Harry Hines Boulevard, Dallas, TX 75390; Deepak.Agrawal@UTSouthwestern.edu

Author and Disclosure Information

Abdulhadi Akhtar, MD
Department of Surgery, University of Texas Southwestern Medical Center, Dallas, TX

Ravindra Sarode, MD
Professor and Chief, Department of Pathology, University of Texas Southwestern Medical Center, Dallas, TX

Deepak Agrawal, MD
Assistant Professor, Department of Internal Medicine, University of Texas Southwestern Medical Center, Dallas, TX

Address: Deepak Agrawal, MD, Department of Internal Medicine, University of Texas Southwestern Medical Center, 5323 Harry Hines Boulevard, Dallas, TX 75390; Deepak.Agrawal@UTSouthwestern.edu

Article PDF
Article PDF
Related Articles

A 43-year-old, previously healthy woman was admitted to the hospital after 1 day of severe epigastric abdominal pain, nausea, and vomiting. She denied alcohol or tobacco use.

Her physical examination revealed normal vital signs and epigastric tenderness without rebound tenderness.

Notable laboratory results:

  • Aspartate aminotransferase 149 U/L (reference range 10–35)
  • Alanine aminotransferase 140 U/L (10–35)
  • Alkaline phosphatase 178 IU/L (35–104)
  • Total bilirubin 1.8 mg/dL (0.2–1.3)
  • Amylase 1,244 U/L (28–100)
  • Lipase 14,628 U/L (7–59).

Abdominal ultrasonography showed a dilated bile duct and gallstones.

The patient was diagnosed with biliary pancreatitis and was treated by placing her on nothing-by-mouth (NPO) status and giving intravenous fluids and analgesics. All symptoms had resolved by hospital day 3. She underwent laparoscopic cholecystectomy and was discharged the following day.

This is a typical case of biliary pancreatitis that was diagnosed and treated appropriately with a positive outcome. But was it necessary or beneficial to measure both the serum amylase and serum lipase to make the correct diagnosis and treat the patient appropriately?

IS MEASURING SERUM AMYLASE NECESSARY?

The American College of Gastroenterology practice guidelines suggest that measuring both serum amylase and serum lipase is not necessary.1 Serum lipase alone is the preferred test for diagnosing acute pancreatitis, since it is more sensitive than serum amylase, just as specific, rises more quickly, and remains elevated longer.

In a retrospective study of 151 patients with acute pancreatitis,2 the sensitivity of lipase was 96.6% and the specificity was 99.4%.2 In contrast, the sensitivity of amylase was 78.6% and the specificity was 99.1%.

In another study,3 in 476 patients with acute pancreatitis, lipase had a sensitivity of 91% vs 62% for amylase. Again, specificity was similar between the two tests (92% for lipase and 93% for amylase). The authors concluded that lipase should replace amylase as the first-line laboratory investigation for suspected acute pancreatitis.

Smith et al4 reviewed 1,825 patients with acute pancreatitis and similarly concluded that pancreatic lipase is a more accurate biomarker of acute pancreatitis than serum amylase.

PRACTICE AT OUR HOSPITAL

Despite this guideline and evidence, concurrent ordering of serum amylase and lipase is common at many institutions.

We evaluated the practice of ordering both serum amylase and lipase for diagnosis of acute pancreatitis at our 300-bed academic medical hospital. From January 2011 through August 2014, our institution completed 26,254 orders for serum amylase and lipase measurement in 13,198 patients. In 9,938 (75%) of the patients, amylase and lipase were ordered concurrently. Of these, 482 patients (4.8%) had either amylase or lipase elevated above the diagnostic threshold, ie, 3 times the upper limit of normal, and 63 of the 482 patients had an elevation in serum amylase greater than 3 times the upper limit of normal without an elevation in serum lipase.

None of the patients had acute pancreatitis clinically (eg, typical abdominal pain, nausea, vomiting) or on imaging (pancreatic edema). The definitive cause of nonpancreatic hyperamylasemia could not be determined in these patients; they did not have evidence of salivary disorder, malignancy, or tubo-ovarian disease, and the hyperamylasemia was believed to be related to renal disease, diabetic ketoacidosis, infection, or medications, or to be idiopathic.

In 12 patients, the discrepancy between an elevated amylase and normal lipase resulted in additional imaging with computed tomography. Four patients were also unnecessarily kept NPO for 1 to 3 days, depriving them of nutrition and prolonging their hospital stay.

To minimize concurrent ordering of serum amylase and lipase, we introduced a best-practice alert in the computerized physician order entry systems. The alert mentioned that “ordering both serum amylase and lipase in cases of suspected pancreatitis is unnecessary. Serum lipase alone is sufficient.” However, ordering providers could still order both tests if they wanted to.

In the 3 months after the alert was implemented, serum lipase was ordered 1,780 times with 532 (30%) concurrent orders of amylase. Before the alert was instituted, amylase testing was ordered a mean of 450 times per month; afterward, this decreased by about 60%.

We are now considering eliminating serum amylase testing, as suggested by prior studies5 and the American Society of Clinical Pathology.6

ELIMINATING NEEDLESS EXPENSES

The relentless and unsustainable rise in healthcare costs has prompted physician-led groups such as the American Board of Internal Medicine Foundation and the American College of Physicians to focus on ways to cut waste and incorporate high-value, cost-conscious care into clinical practice.

In 2009 alone, waste in total healthcare expenditures was estimated at $765 billion. More than half of this astronomical figure was attributed to unnecessary and inefficiently delivered services, expenditures that physicians can directly avoid with changes to their practice.7,8 Unnecessary laboratory tests such as serum amylase are just one of many wasteful practices.

Hospitals have much to lose when unnecessary tests are ordered. For inpatient hospital admissions in the United States, payment is based on the diagnosis-related group system, in which hospitals are paid a fixed amount per diagnosis. There is no additional reimbursement for laboratory tests. An unnecessary test such as serum amylase in suspected cases of acute pancreatitis thus becomes an expense with no corresponding benefit.

The cost of performing a serum amylase test for a typical laboratory is around $4 to $6. Serum amylase testing at our hospital resulted in unnecessary expense of about $35,000 annually. If we add the costs of additional imaging and prolonged hospitalization, the expenses are substantially more.

Despite this, most hospitals have been unwilling or unable to tackle the problem. This may be due to respect for physician autonomy, seemingly small financial loss, or organizational inertia. For the entire healthcare system, these seemingly minor costs add up. For example, from 2011 to 2014, Medicare Part B alone spent $19.4 million on serum amylase testing.

Ordering unnecessary laboratory tests is not a problem specific to our hospital, but rather a common problem encountered at many hospitals. Recognizing the widespread practice of ordering amylase, the Choosing Wisely initiative shared new recommendations from the American Society for Clinical Pathology supporting the use of lipase instead of amylase in suspected acute pancreatitis.7

Physicians who continue to order these tests show a disregard for evidence-based medicine, patient care, and healthcare costs.

CLINICAL BOTTOM LINE

Concurrent use of amylase and lipase testing to diagnose acute pancreatitis is an unnecessary expense for the hospital and can negatively affect patient care as it can lead to additional tests and prolonged hospitalization. Steps should be taken to minimize ordering of amylase by educating physicians and instituting best-practice alerts, or by eliminating the test altogether.

A 43-year-old, previously healthy woman was admitted to the hospital after 1 day of severe epigastric abdominal pain, nausea, and vomiting. She denied alcohol or tobacco use.

Her physical examination revealed normal vital signs and epigastric tenderness without rebound tenderness.

Notable laboratory results:

  • Aspartate aminotransferase 149 U/L (reference range 10–35)
  • Alanine aminotransferase 140 U/L (10–35)
  • Alkaline phosphatase 178 IU/L (35–104)
  • Total bilirubin 1.8 mg/dL (0.2–1.3)
  • Amylase 1,244 U/L (28–100)
  • Lipase 14,628 U/L (7–59).

Abdominal ultrasonography showed a dilated bile duct and gallstones.

The patient was diagnosed with biliary pancreatitis and was treated by placing her on nothing-by-mouth (NPO) status and giving intravenous fluids and analgesics. All symptoms had resolved by hospital day 3. She underwent laparoscopic cholecystectomy and was discharged the following day.

This is a typical case of biliary pancreatitis that was diagnosed and treated appropriately with a positive outcome. But was it necessary or beneficial to measure both the serum amylase and serum lipase to make the correct diagnosis and treat the patient appropriately?

IS MEASURING SERUM AMYLASE NECESSARY?

The American College of Gastroenterology practice guidelines suggest that measuring both serum amylase and serum lipase is not necessary.1 Serum lipase alone is the preferred test for diagnosing acute pancreatitis, since it is more sensitive than serum amylase, just as specific, rises more quickly, and remains elevated longer.

In a retrospective study of 151 patients with acute pancreatitis,2 the sensitivity of lipase was 96.6% and the specificity was 99.4%.2 In contrast, the sensitivity of amylase was 78.6% and the specificity was 99.1%.

In another study,3 in 476 patients with acute pancreatitis, lipase had a sensitivity of 91% vs 62% for amylase. Again, specificity was similar between the two tests (92% for lipase and 93% for amylase). The authors concluded that lipase should replace amylase as the first-line laboratory investigation for suspected acute pancreatitis.

Smith et al4 reviewed 1,825 patients with acute pancreatitis and similarly concluded that pancreatic lipase is a more accurate biomarker of acute pancreatitis than serum amylase.

PRACTICE AT OUR HOSPITAL

Despite this guideline and evidence, concurrent ordering of serum amylase and lipase is common at many institutions.

We evaluated the practice of ordering both serum amylase and lipase for diagnosis of acute pancreatitis at our 300-bed academic medical hospital. From January 2011 through August 2014, our institution completed 26,254 orders for serum amylase and lipase measurement in 13,198 patients. In 9,938 (75%) of the patients, amylase and lipase were ordered concurrently. Of these, 482 patients (4.8%) had either amylase or lipase elevated above the diagnostic threshold, ie, 3 times the upper limit of normal, and 63 of the 482 patients had an elevation in serum amylase greater than 3 times the upper limit of normal without an elevation in serum lipase.

None of the patients had acute pancreatitis clinically (eg, typical abdominal pain, nausea, vomiting) or on imaging (pancreatic edema). The definitive cause of nonpancreatic hyperamylasemia could not be determined in these patients; they did not have evidence of salivary disorder, malignancy, or tubo-ovarian disease, and the hyperamylasemia was believed to be related to renal disease, diabetic ketoacidosis, infection, or medications, or to be idiopathic.

In 12 patients, the discrepancy between an elevated amylase and normal lipase resulted in additional imaging with computed tomography. Four patients were also unnecessarily kept NPO for 1 to 3 days, depriving them of nutrition and prolonging their hospital stay.

To minimize concurrent ordering of serum amylase and lipase, we introduced a best-practice alert in the computerized physician order entry systems. The alert mentioned that “ordering both serum amylase and lipase in cases of suspected pancreatitis is unnecessary. Serum lipase alone is sufficient.” However, ordering providers could still order both tests if they wanted to.

In the 3 months after the alert was implemented, serum lipase was ordered 1,780 times with 532 (30%) concurrent orders of amylase. Before the alert was instituted, amylase testing was ordered a mean of 450 times per month; afterward, this decreased by about 60%.

We are now considering eliminating serum amylase testing, as suggested by prior studies5 and the American Society of Clinical Pathology.6

ELIMINATING NEEDLESS EXPENSES

The relentless and unsustainable rise in healthcare costs has prompted physician-led groups such as the American Board of Internal Medicine Foundation and the American College of Physicians to focus on ways to cut waste and incorporate high-value, cost-conscious care into clinical practice.

In 2009 alone, waste in total healthcare expenditures was estimated at $765 billion. More than half of this astronomical figure was attributed to unnecessary and inefficiently delivered services, expenditures that physicians can directly avoid with changes to their practice.7,8 Unnecessary laboratory tests such as serum amylase are just one of many wasteful practices.

Hospitals have much to lose when unnecessary tests are ordered. For inpatient hospital admissions in the United States, payment is based on the diagnosis-related group system, in which hospitals are paid a fixed amount per diagnosis. There is no additional reimbursement for laboratory tests. An unnecessary test such as serum amylase in suspected cases of acute pancreatitis thus becomes an expense with no corresponding benefit.

The cost of performing a serum amylase test for a typical laboratory is around $4 to $6. Serum amylase testing at our hospital resulted in unnecessary expense of about $35,000 annually. If we add the costs of additional imaging and prolonged hospitalization, the expenses are substantially more.

Despite this, most hospitals have been unwilling or unable to tackle the problem. This may be due to respect for physician autonomy, seemingly small financial loss, or organizational inertia. For the entire healthcare system, these seemingly minor costs add up. For example, from 2011 to 2014, Medicare Part B alone spent $19.4 million on serum amylase testing.

Ordering unnecessary laboratory tests is not a problem specific to our hospital, but rather a common problem encountered at many hospitals. Recognizing the widespread practice of ordering amylase, the Choosing Wisely initiative shared new recommendations from the American Society for Clinical Pathology supporting the use of lipase instead of amylase in suspected acute pancreatitis.7

Physicians who continue to order these tests show a disregard for evidence-based medicine, patient care, and healthcare costs.

CLINICAL BOTTOM LINE

Concurrent use of amylase and lipase testing to diagnose acute pancreatitis is an unnecessary expense for the hospital and can negatively affect patient care as it can lead to additional tests and prolonged hospitalization. Steps should be taken to minimize ordering of amylase by educating physicians and instituting best-practice alerts, or by eliminating the test altogether.

References
  1. Banks PA, Freeman ML, Practice Parameters Committee of the American College of Gastroenterology. Practice guidelines in acute pancreatitis. Am J Gastroenterol 2006; 101:2379–2400.
  2. Gomez D, Addison A, De Rosa A, Brooks A, Cameron IC. Retrospective study of patients with acute pancreatitis: is serum amylase still required? BMJ Open 2012; 2. pii:e001471.
  3. Hofmeyr S, Meyer C, Warren BL. Serum lipase should be the laboratory test of choice for suspected acute pancreatitis. S Afr J Surg 2014; 52:72–75.
  4. Smith RC, Southwell-Keely J, Chesher D. Should serum pancreatic lipase replace serum amylase as a biomarker of acute pancreatitis? ANZ J Surg 2005; 75:399–404.
  5. Volz KA, McGillicuddy DC, Horowitz GL, Wolfe RE, Joyce N, Sanchez LD. Eliminating amylase testing from the evaluation of pancreatitis in the emergency department. West J Emerg Med 2010; 11:344–347.
  6. American Society for Clinical Pathology. Do not test for amylase in cases of suspected pancreatitis. Instead, test for lipase. Choosing Wisely; 2016. www.choosingwisely.org/clinician-lists/american-society-clinical-pathology-testing-for-amylase. Accessed August 3, 2017.
  7. Smith M, Saunders R, Stuckhardt L, McGinnis JM; Committee on the Learning Health Care System in America, eds. Best Care at Lower Cost: The Path to Continuously Learning Health Care in America. Washington, DC: The National Academies Press; 2013. www.hep.fsu.edu/~wahl/artic/NAP/HealthCare13444.pdf. Accessed August 3, 2017.
  8. American College of Physicians. Eliminating healthcare waste and overordering of tests. www.acponline.org/clinical-information/high-value-care/medical-educators-resources/curriculum-for-educators-and-residents/curriculum-version-3. Accessed August 3, 2017.
References
  1. Banks PA, Freeman ML, Practice Parameters Committee of the American College of Gastroenterology. Practice guidelines in acute pancreatitis. Am J Gastroenterol 2006; 101:2379–2400.
  2. Gomez D, Addison A, De Rosa A, Brooks A, Cameron IC. Retrospective study of patients with acute pancreatitis: is serum amylase still required? BMJ Open 2012; 2. pii:e001471.
  3. Hofmeyr S, Meyer C, Warren BL. Serum lipase should be the laboratory test of choice for suspected acute pancreatitis. S Afr J Surg 2014; 52:72–75.
  4. Smith RC, Southwell-Keely J, Chesher D. Should serum pancreatic lipase replace serum amylase as a biomarker of acute pancreatitis? ANZ J Surg 2005; 75:399–404.
  5. Volz KA, McGillicuddy DC, Horowitz GL, Wolfe RE, Joyce N, Sanchez LD. Eliminating amylase testing from the evaluation of pancreatitis in the emergency department. West J Emerg Med 2010; 11:344–347.
  6. American Society for Clinical Pathology. Do not test for amylase in cases of suspected pancreatitis. Instead, test for lipase. Choosing Wisely; 2016. www.choosingwisely.org/clinician-lists/american-society-clinical-pathology-testing-for-amylase. Accessed August 3, 2017.
  7. Smith M, Saunders R, Stuckhardt L, McGinnis JM; Committee on the Learning Health Care System in America, eds. Best Care at Lower Cost: The Path to Continuously Learning Health Care in America. Washington, DC: The National Academies Press; 2013. www.hep.fsu.edu/~wahl/artic/NAP/HealthCare13444.pdf. Accessed August 3, 2017.
  8. American College of Physicians. Eliminating healthcare waste and overordering of tests. www.acponline.org/clinical-information/high-value-care/medical-educators-resources/curriculum-for-educators-and-residents/curriculum-version-3. Accessed August 3, 2017.
Issue
Cleveland Clinic Journal of Medicine - 84(9)
Issue
Cleveland Clinic Journal of Medicine - 84(9)
Page Number
670-672
Page Number
670-672
Publications
Publications
Topics
Article Type
Display Headline
Measuring both serum amylase and lipase for acute pancreatitis lowers quality and raises cost
Display Headline
Measuring both serum amylase and lipase for acute pancreatitis lowers quality and raises cost
Legacy Keywords
amylase, lipase, laboratory testing, smart testing, pancreatitis, Abdulhadi Akhtar, Ravindra Sarode, Deepak Agrawal
Legacy Keywords
amylase, lipase, laboratory testing, smart testing, pancreatitis, Abdulhadi Akhtar, Ravindra Sarode, Deepak Agrawal
Sections
Disallow All Ads
Content Gating
No Gating (article Unlocked/Free)
Alternative CME
Disqus Comments
Default
Article PDF Media